Content Knowledge: 5038

Ace your homework & exams now with Quizwiz!

Question:Which of the following components of a citation is unique to electronically accessed sources? A. The date the source was accessed B. The place of publication of the accessed source C. The publication date of the accessed source D. The title of the accessed source

A. The date the source was accessed Option (A) is correct. This question tests your ability to recognize appropriate research methods. The date a source was accessed should be included only when citing an electronic source. The date of access is not a necessary inclusion when citing a print source.

Him the Almighty Power Hurled headlong flaming from the ethereal sky, With hideous ruin and combustion, down To bottomless perdition; there to dwell In adamantine chains and penal fire, Who durst defy the Omnipotent to arms. In line 6, "Who durst defy the Omnipotent to arms" modifies A."Him" (line 1) B."Almighty Power" (line 1) C."ethereal sky" (line 2) D."penal fire" (line 5)

A."Him" (line 1) Option (A) is correct. This question tests your ability to understand the syntax of the poem. Line 6 modifies "Him" in line 1. "Him" refers to Satan in the Judeo-Christian belief system; it is Satan who challenges the authority of "the Almighty Power," starts a war in heaven (line 6), and is hurled from heaven to hell (lines 1-5).

A hush is over all the teeming lists, And there is pause, a breath-space in the strife; A spirit brave has passed beyond the mists And vapors that obscure the sun of life. And Ethiopia, with bosom torn, Laments the passing of her noblest born. For her his voice, a fearless clarion, rung That broke in warning on the ears of men; For her the strong bow of his power he strung, And sent his arrows to the very den Where grim Oppression held his bloody place And gloated o'er the mis'ries of a race. Through good and ill report he cleaved his way Right onward, with his face set toward the heights, Nor feared to face the foeman's dread array,— The lash of scorn, the sting of petty spites. He dared the lightning in the lightning's track, And answered thunder with his thunder back. —Paul Laurence Dunbar, from "Frederick Douglass" A. Student A: This poem is about war. It's about "strife" or "bloody" fighting. The champion "cleaved his way Right onward." B. Student B: A famous leader is praised for all the contributions he made to his people. He met much oppression in his struggle for his people but overcame it all. C. Student C: This lady, Ethiopia, is very sad when someone close to her dies. This person, who was killed in war, helped people. He must have been some kind of a super warrior or maybe even a god. D. Student D: It interested me the way this poem is so concerned with sound and noise. It begins on a quiet note and death but then builds up to the final stanza which is full of sound words and action. "What often misleads less able students when they read poetry is the very commonness of some words. Most will look up words they have never seen before; what deceives them are words that they 'know' perfectly well, but 'know' in an inappropriate sense." Question:In which of the following lines from Dunbar's poem is the underlined word likely to present the kind of problem discussed above? A.A hush is over all the teeming lists (line 1) B.And there is pause, a breath-space in the strife (line 2) C.A spirit brave has passed beyond the mists (line 3) D.And vapors that obscure the sun of life (line 4)

A.A hush is over all the teeming lists (line 1) Option (A) is correct. This question asks you to evaluate the use of words in a poem. The word "lists" is likely to present a problem in comprehension to some students because these students will know the modern meaning of the word, but will not realize that the word also has a less common, older meaning that is, in fact, more appropriate within the context of the poem. The most common meaning of "lists" is "records or registers of names or items," but "lists" also can denote, as here, the grounds or arena used for combat or competition, a meaning dating from medieval times and practices.

Read the following student writing sample and then answer the question below. I was lucky enough to go to my grandpa's birthday party this past weekend. At 84 years old, he is still throwing himself fabulous birthday parties. I am amazed at his capacity for happiness and joy, especially since he has had more than his share of grief in his life. Despite hardships he has gone through, he still makes the choice to find joy in the little things. Spending time with him reminds me that happiness is always a choice. I also got to go see a play that I have been dying to see ever since I heard about it. I was surprised to see the same theme playing out on the stage - happiness is a matter of choice, not a condition that a person either has or doesn't. Question:The excerpt is best described as which type of writing? A.A journal entry B.A narrative essay C.A persuasive speech D.An informative letter

A.A journal entry Option (A) is correct. This question tests your ability to identify examples of common modes of writing. The excerpt contains personal information and reflections that typify journal writing.

Over the past few decades, the health of American students has greatly declined. Young adults are increasingly being diagnosed with health problems that used to be more typical of people over 50, such as blood pressure issues, type 2 diabetes, and obesity. Experts point to changes in diet and declining levels of physical activity as contributing factors. Since young people spend so much of their time at schools, many experts believe schools have an obligation to address this issue. Propose a course of action for your school to take in order to address this health issue. Explain why the school should address the issue and why the school should choose your course of action over other possible solutions. Be sure that you address your writing to the appropriate audience that has the power to make decisions about the issue. Which of the following writing formats is most appropriate for allowing students to share their writing with an audience that has the power to bring about change? A.A letter B.An essay C.A blog entry D.A newspaper article

A.A letter Option (A) is correct. This question tests your ability to identify the most appropriate writing format for a given task. The prompt directs students to address their writing to an audience that has the authority to make decisions about the topic. Although a newspaper article would be an effective means of communicating the issue and proposed solution, newspaper articles are not addressed to a specific audience.

My sister, Mrs. Joe Gargery, was more than twenty years older than I, and had established a great reputation with herself and her neighbours because she had brought me up "by hand." Having at that time to find out for myself what the expression meant, and knowing her to have a hard and heavy hand, and to be much in the habit of laying it upon her husband as well as upon me, I supposed that Joe Gargery and I were both brought up by hand. She was not a good-looking woman, my sister, and I had a general impression that she must have made Joe Gargery marry her by hand. Joe was a fair man, with curls of flaxen hair on each side of his smooth face, and with eyes of such a very undecided blue that they seemed to have somehow got mixed with their own whites. He was a mild, good-natured, sweet-tempered, easy-going, foolish, dear fellow—a sort of Hercules in strength and also in weakness. —Charles Dickens, from Great Expectations Which of the following is used in the description of Joe Gargery in the second paragraph? A.Allusion B.Simile C.Metonymy D.Irony

A.Allusion Option (A) is correct. This question tests your knowledge of literary terms. In the passage's final line, the narrator directly compares Joe to Hercules, the powerful demigod from Greco-Roman mythology, who was also overcome by a determined woman.

Ms. Dunn is having her eighth-grade students participate in whole-class discussions focused on current events. At the beginning of the class, she has the students read a short article, and then she initiates the discussion. She notices that the students are having difficulty referring back to the article during their conversation. Which of the following should Ms. Dunn instruct her students to do while reading in order to promote active participation? A.Annotate the article with questions and comments B.Read the article aloud instead of reading it silently C.Memorize facts about the article's author D.Rewrite the article in their own words

A.Annotate the article with questions and comments Option (A) is correct. Annotating an article promotes metacognition and enables students to easily find and refer back to important parts of the text.

(1) People have debated the best ways to educate children for decades. (2) One of the many aspects of education that is often debated is the structure of the school years. (3) Historically, students were given summers off so they could help their families harvest crops. (4) As more families got away from farming, this became less necessary, but many argued that money is saved by closing schools for the summer. (5) With other countries surpassing the test scores of American students, the concept of year-round schooling is again being pushed by lawmakers. Which of the following is the best revision of sentence 5 ? A.Because students in other countries are surpassing the test scores of American students, lawmakers are again pushing the concept of year-round school. B.With other countries performing better on tests than American students, the concept of year-round school is again being pushed by lawmakers. C.Other countries are surpassing the test scores of American students, and so lawmakers are again pushing the concept of year-round school. D.Children in other countries are scoring higher on tests than children in the United States, which is concerning lawmakers who are pushing for year-round school.

A.Because students in other countries are surpassing the test scores of American students, lawmakers are again pushing the concept of year-round school. Option (A) is correct. This question tests your facility with writing conventions. Of all the options, only (A) is grammatically correct and eliminates the misplaced modifier and passive voice.

At the beginning of the school year, Mr. Martin gives his students a reading survey in which they are asked to share topics they want to learn more about, their genre preferences, and the names of the most difficult books they have read. For which of the following tasks will the information gathered from the survey be most helpful to Mr. Martin? A.Choosing texts that are based on student interests and abilities B.Determining reading strategies that will be taught that year C.Identifying students who will need interventions to read at grade level D.Selecting curriculum units that can be compacted based on students' prior knowledge

A.Choosing texts that are based on student interests and abilities Option (A) is correct. This question tests your knowledge of the instructional uses of one kind of student-generated information. The information collected on the survey would allow Mr. Martin to choose texts that are based on ability and interest. This approach ensures that selected texts will be within students' instructional level and that they will reflect topics of interest to the students. The best answer is (A).

Question:Which of the following cognates is most different in meaning and usage from its original Latin root? A.Corporate B.Corpulent C.Corpse D.Corporeal

A.Corporate Option (A) is correct. This question asks you to recognize the manifestation of etymological development. These cognates share the Latin root corpus, or "body." All of the current uses connote some aspect of physical body except "corporate." This word has come to mean "united or combined into one collective" and has thus evolved to a meaning more different from its root than the three other choices.

A teacher displays the following sentence from an informational essay and then asks students which word in the parentheses is the best choice for the sentence. Each restaurant in the district is (noted or notorious) for its mouthwatering cuisine. Question:In the activity, the teacher is focusing students on which of the following? A.Denotation B.Annotation C.Figurative language D.Sentence structure

A.Denotation Option (A) is correct. This question tests your ability to recognize how word choice impacts meaning. Denotation is the definition of a word. Something that is noted is well-known and celebrated, with a positive reputation. Something that is notorious is widely and unfavorably known. The word choice, therefore, impacts the meaning of the sentence.

In the saddest prefatory note I know of, Coleridge says that he dreamed a poem four or five times as long as the "Kubla Khan" we now have. Awakening, he hurried to write it down, but managed only fifty-four lines before "he was unfortunately called out by a person on business from Porlock." On regaining his room, he was dismayed to find that, "with the exception of eight or ten scattered lines and images, all the rest had passed away like the images on the surface of a stream into which a stone has been cast, but, alas! without the after restoration of the latter!" Question:How is supporting evidence primarily presented in the passage? A.Direct quotation from a primary source B.Indirect quotation via a secondary source C.Paraphrasing the views of a single person D.Summarizing the argument of an authority

A.Direct quotation from a primary source Option (A) is correct. This question asks you to identify how supporting evidence is presented in the passage. The purpose of the passage is to describe an event in Coleridge's life. The key details of the story are provided by direct quotations from the primary source of the anecdote; i.e., Coleridge.

Mr. Marcs is planning a study unit on novels for his English class. Jonathan, an English-language learner, has recently graduated from his ESL program and has been placed into Mr. Marcs' class. Question:Which of the following is most likely to create a comfortable environment for Jonathan to actively participate in the class? Select all that apply. A.Displaying teacher-made posters in the classroom that vividly depict the characters, settings, and imagery from the novel currently being discussed B.Writing student names on sticks and then pulling out the names to determine who is going to participate in discussions, instead of calling on students who have raised their hand C.Preparing teacher-made outlines of the chapters that will be discussed and then distributing the outlines to the entire class D.Writing the daily discussion question on the board and then requiring each student to respond to the question orally E.Assigning students a text-to-self-reflection journal to complete after reading each chapter of the novel and then allowing them to refer to it during discussions

A.Displaying teacher-made posters in the classroom that vividly depict the characters, settings, and imagery from the novel currently being discussed C.Preparing teacher-made outlines of the chapters that will be discussed and then distributing the outlines to the entire class E.Assigning students a text-to-self-reflection journal to complete after reading each chapter of the novel and then allowing them to refer to it during discussions Options (A), (C), and (E) are correct. This question tests your ability to identify strategies that would create a safe environment for an English-language learner (ELL). Students who have been able to find success in interpersonal communication often have trouble transferring those communication skills to an academic situation. Research has shown that providing visual representation of content, providing outlines of second language text, and allowing students to make connections to their own culture helps to support the ELL in assimilating into the classroom. Requiring ELLs to participate in discussions before they are ready can have an adverse effect on student adjustment.

A high school ELA teacher creates the checklist below based on commonly observed weaknesses in student writing. Writers Check List __ Sentences begin in different ways. __ Sentences build on the ones that precede them. __ Sentences vary in length. __ The meaning of each sentence is clear. __ Sentences flow and are grammatically correct. __ There are no run-on sentences. __ Sentences are complete. It would be most appropriate to introduce the checklist to students when modeling which of the following components of the writing process? A.Editing B.Drafting C.Prewriting D.Publishing

A.Editing Option (A) is correct. This question tests your ability to recognize characteristics of steps in the writing process. The checklist addresses details that should be examined during the editing component of the writing process. During editing, a written draft is examined for clarity and grammatical correctness. The goal is to improve the readability of the work before it is published.

A. For thou hast sent her a mantle of green, As green as any grass, And bade her come to the silver wood To hunt with Child Maurice. B. An Ace of Hearts steps forth: the King unseen Lurked in her hand, and mourned his captive Queen: He springs to vengeance with an eager pace, And falls like thunder on the prostrate Ace. The nymph exulting fills with shouts the sky; The walls, the woods, and long canals reply. C. Who would have thought my shriveled heart Could have recovered greenness? It was gone Quite underground; as flowers depart To see their mother-root, when they have blown, Were they together All the hard weather, Dead to the world, keep house unknown. D. But wherefore rough, why cold and ill at ease? Aha, that is a question! Ask, for that, What knows,—the something over Setebos That made Him, or He, may be, found and fought, Worsted, drove off and did to nothing, perchance. Which is an example of the ballad stanza? A.Excerpt A B.Excerpt B C.Excerpt C D.Excerpt D

A.Excerpt A Option (A) is correct. This question tests your knowledge of literary genres. Ballads are anonymous narrative poems; the ballad stanza is a four-line stanza of alternating tetrameter and trimeter lines with a rhyme of abab or, as in (A), from the English ballad "Child Maurice," abcb.

I was walking by the Thames. Half-past morning on an autumn day. Sun in a mist. Like an orange in a fried-fish shop. All bright below. Low tide, dusty water and a crooked bar of straw, chicken-boxes, dirt and oil from mud to mud. Like a viper swimming in skim milk. The old serpent, symbol of nature and love. —Joyce Cary, from The Horse's Mouth Question:Which of the following best describes the descriptive strategy used most consistently in the excerpt? A.Fragments connecting a series of perceptions and reactions B.Fragments connecting a series of actions and events C.Fragments connected by water imagery D.Fragments connected by color imagery

A.Fragments connecting a series of perceptions and reactions Option (A) is correct. This question tests your ability to identify descriptive patterns and structures and their effects. In this passage, which begins the novel, the author presents a series of quick perceptions of and reactions to the River Thames on an autumn morning. Most of the clauses or phrases that end with periods are actually sentence fragments. The sequence of fragments — "Sun in a mist. Like an orange in a fried-fish shop. All bright below," and so on — builds up the total scene.

After the school board announced that it would be ending instrumental music programs in the school, the high school's English teachers asked their students to share their reactions to the announcement with the editor of the school newspaper. Which of the following criteria should the students consider when choosing the most appropriate format for their writing? Select all that apply. A.Intended audience B.Purpose C.Length D.Bias E.Task

A.Intended audience B.Purpose E.Task Options (A), (B), and (E) are correct. This question tests your ability to recognize the impact that different elements have on effective writing. The intended audience determines the point of view, tone, and format of the piece. Therefore, (A) is correct. Purpose drives the writing, and one's purpose can only be achieved by presenting the material in the most appropriate way; that is, choosing the best format. Therefore, (B) is correct. The task is to respond to the announcement made by the school board, and, therefore, the content of the writing piece should be directed toward the school board. Thus, option (E) is correct.

Which of the following sentences is grammatically correct? A.It's time for the children to take their nap. B.Jumping off the cliff, the crowd was thrilled by the diver. C.Everybody with tickets are supposed to use the door on the left. D.Please keep this discussion between you and I.

A.It's time for the children to take their nap. Option (A) is correct. This question requires you to apply your knowledge of grammar. (B) is misconstructed in that the initial phrase illogically modifies "the crowd," the subject of a passive construction. In (C), the verb should be "is" — not "are" — to agree with the singular subject "Everybody." In (D), the object of the preposition "between" should be "you and me." Every member of the group of children takes one nap, so (A) is grammatically correct, making it the correct answer.

Which of the following is a small-group discussion strategy? A.Jigsaw B.KWL C.Reciprocal reading D.SQ3R

A.Jigsaw Option (A) is correct. This question tests your ability to identify a variety of techniques for encouraging productive participation and active listening in discussions. Jigsaw is a method of small-group discussion in which small groups of students read a certain portion of an assigned text and teach its contents to other small group of students. Though the other choices are effective teaching strategies, they do not directly support effective discussion.

During the Second World War, Japanese Americans who lived in the United States were sent to internment camps until the war was over. In her memoir Farewell to Manzanar, Jeanne Wakatsuki tells the story of her internment at one of the camps, Manzanar. The excerpt below describes her first observations upon arriving at the camp with her family at age seven. (1) I could see a few tents set up, the first rows of black barracks, and beyond them, blurred by sand, rows of barracks that seemed to spread for miles across this plain. (2) People were sitting on cartons or milling around, with their backs to the wind, waiting, to see which friends or relatives might be on this bus. (3) As we approached, they turned or stood up, and some moved toward us expectantly. (4) But inside the bus no one stirred. (5) No one waved or spoke. (6) They just stared out the windows, ominously silent. (7) I didn't understand this. (8) Hadn't we finally arrived, our whole family intact? (9) I opened a window, leaned out, and yelled happily, "Hey! This whole bus is full of Wakatsukis!" Question:The excerpt from Farewell to Manzanar represents which of the following modes of writing? A.Narrative B.Argumentative C.Descriptive D.Expository

A.Narrative Option (A) is correct. This question tests your ability to recognize different modes of writing. Farewell to Manzanar is a memoir. Memoir is a form of narrative writing, which is distinguished by focusing on one or more significant life events of the writer.

The epic Beowulf and its shorter elegiac contemporaries, "The Wanderer" and "The Seafarer," were all originally written in what language? A.Old English B.Middle English C.Elizabethan English D.Early Modern English

A.Old English Option (A) is correct. The question tests your knowledge of English language history. Beowulf and other Anglo-Saxon works were written in Old English, a Germanic language that gave us some of our most basic, everyday words (e.g., "father," "give," and "day").

A person who drives an older car would do well to keep a set of jumper cables in the trunk, in the event that your car suddenly refuses to start up. Question:Which of the following is a grammatical error found in the sentence above? A.Pronoun shift B.Misplaced modifier C.Subject-verb agreement D.Run-on sentence

A.Pronoun shift Option (A) is correct. A pronoun shift occurs when a writer begins by using a particular type of pronoun and then shifts abruptly to using a different type of pronoun. In the sentence shown above, the writer begins by discussing "a person who drives an older car," then shifts to talking about "your car."

_________. A wreck might be valuable for itself, of course, rather than for its cargo or the scrap value of its hull and fittings. A ship like the Mary Rose is of historical interest, and her salvaging was more in the nature of an archaeological dig that happened not to be performed on dry land. The Titanic, too, is now viewed in a similar light, thanks mainly to some very well orchestrated publicity intended to supplant the ship's poetic and melancholy status with that of a treasure trove of artifacts. Question:Which of the following would make the most appropriate first sentence for the paragraph? A.Salvagers do not bother to recover a wreck unless it involves something of great enough value to make the venture worthwhile. B.What really grabs people's attention is the prospect of large ships being found and restored to the upper air. C.Newspaper editors understand perfectly that treasure and treasure hunts come very high on the list of things that induce their readers to suspend disbelief. D.The principles of researching and locating remain common to both shallow and deepwater wrecks, but the techniques are mostly quite different.

A.Salvagers do not bother to recover a wreck unless it involves something of great enough value to make the venture worthwhile. Option (A) is correct. The question asks you to identify the most appropriate topic sentence for the passage. The passage is primarily concerned with discussing the characteristics that make different shipwrecks attractive to salvagers. (A) states that salvagers will not recover a wreck unless it is of great enough value to make the venture worthwhile. Therefore, (A) is the answer.

My mistress' eyes are nothing like the sun;Coral is far more red than her lips red;If snow be white, why then her breasts are dun;If hairs be wires, black wires grow on her head.I have seen roses damasked, red and white,But no such roses see I in her cheeks;And in some perfumes is there more delightThan in the breath that from my mistress reeks.I love to hear her speak, yet well I knowThat music hath a far more pleasing sound;I grant I never saw a goddess go;My mistress, when she walks, treads on the ground.And yet by heaven, I think my love as rareAs any she belied with false compare. Question:The poem is written in which of the following poetic forms? A.Sonnet B.Ode C.Ballad D.Elegy

A.Sonnet Option (A) is correct. The poem follows all the rules of a sonnet. It is fourteen lines in length, has the requisite rhyme scheme, and is written in iambic pentameter.

Read the following two statements. 1. I'd rather do yoga than run. 2. I'd rather do yoga, then run. Question:What do the two statements mean? A.Statement 1 means that the writer would prefer to do yoga over running, and statement 2 means that the writer wants to do yoga first and run later. B.Statement 1 means that the writer wants to do yoga first and run later, and statement 2 means that the writer would prefer to do yoga over running. C.Statement 1 means that the writer would prefer to run over doing yoga, and statement 2 means that the writer wants to run first and do yoga later. D.Statement 1 means that the writer wants to run first and do yoga later, and statement 2 means that the writer would prefer to run over doing yoga.

A.Statement 1 means that the writer would prefer to do yoga over running, and statement 2 means that the writer wants to do yoga first and run later. Option (A) is correct. This question tests your ability to understand the conventions of standard English usage. Because "than" indicates comparison, the first statement means that the writer prefers yoga to running. "Then" signifies time, so the second statement means that the writer wants to practice yoga first and run later.

Question:Which of the following describes an appropriate peer-review activity? A.Students evaluate each other's written drafts and offer suggestions for revision. B.Students evaluate a classmate's entire writing portfolio and assign a grade. C.Students select a fellow classmate's graded writing assignment and rewrite it. D.Students critique the teacher's model response to the assignment.

A.Students evaluate each other's written drafts and offer suggestions for revision. Option (A) is correct. The question tests your understanding of writing evaluation. To be a peer editor is to study a fellow classmate's writing and to discover how well the writer is achieving his or her goals. Put another way, the job of the peer editor is to offer encouraging comments and constructive criticism to fellow writers so that they may improve upon what they have already started.

A hush is over all the teeming lists, And there is pause, a breath-space in the strife; A spirit brave has passed beyond the mists And vapors that obscure the sun of life. And Ethiopia, with bosom torn, Laments the passing of her noblest born. For her his voice, a fearless clarion, rung That broke in warning on the ears of men; For her the strong bow of his power he strung, And sent his arrows to the very den Where grim Oppression held his bloody place And gloated o'er the mis'ries of a race. Through good and ill report he cleaved his way Right onward, with his face set toward the heights, Nor feared to face the foeman's dread array,— The lash of scorn, the sting of petty spites. He dared the lightning in the lightning's track, And answered thunder with his thunder back. —Paul Laurence Dunbar, from "Frederick Douglass" A. Student A: This poem is about war. It's about "strife" or "bloody" fighting. The champion "cleaved his way Right onward." B. Student B: A famous leader is praised for all the contributions he made to his people. He met much oppression in his struggle for his people but overcame it all. C. Student C: This lady, Ethiopia, is very sad when someone close to her dies. This person, who was killed in war, helped people. He must have been some kind of a super warrior or maybe even a god. D. Student D: It interested me the way this poem is so concerned with sound and noise. It begins on a quiet note and death but then builds up to the final stanza which is full of sound words and action. Question:To which of the following should a student be referred for the fullest discussion of a word that presents the kind of problem discussed in the previous question? A.The Oxford English Dictionary B.Baugh's History of the English Language C.The Random House Dictionary of the English Language D.Webster's New Collegiate Dictionary

A.The Oxford English Dictionary Option (A) is correct. This question tests your knowledge of reference works for English. Of the references listed, The Oxford English Dictionary would be the most appropriate for a student to consult. The Oxford English Dictionary gives not only the current, common meanings of a word, but also a history of the different ways a word has been used since its recorded entry into the language. A. C. Baugh's Baugh's History of the English Language is unlikely to treat in depth the meanings of individual words from Dunbar's poem. While dictionaries such as The Random House Dictionary of the English Language and Webster's New Collegiate Dictionary often present some of the historical background of a word, neither gives as full a treatment as does The Oxford English Dictionary.

A slumber did my spirit seal; I had no human fears: She seemed a thing that could not feel The touch of earthly years. No motion has she now, no force; She neither hears nor sees; Rolled round in earth's diurnal course, With rocks, and stones, and trees. —William Wordsworth, "A Slumber Did My Spirit Seal" Question:Which of the following statements best describes a literary device used in the poem? A.The poem uses alliteration to create a somber mood. B.The poem uses the rhyme scheme of a sonnet. C.The poem uses personification to portray Earth as a woman. D.The poem is written in iambic pentameter.

A.The poem uses alliteration to create a somber mood. Option (A) is correct. This question tests your understanding of how poetic devices and structures contribute to meaning. The poem is about the death of a person. Repeated alliterative use of "s" (slumber, spirit, seal, she) and "r" (rolled, round, rocks) help to create a somber, melancholy mood.

Over the past few decades, the health of American students has greatly declined. Young adults are increasingly being diagnosed with health problems that used to be more typical of people over 50, such as blood pressure issues, type 2 diabetes, and obesity. Experts point to changes in diet and declining levels of physical activity as contributing factors. Since young people spend so much of their time at schools, many experts believe schools have an obligation to address this issue. Propose a course of action for your school to take in order to address this health issue. Explain why the school should address the issue and why the school should choose your course of action over other possible solutions. Be sure that you address your writing to the appropriate audience that has the power to make decisions about the issue. Question:Which of the following is the most important factor to consider when determining the format of the writing piece? A.The ways in which the audience expects to receive communications B.The writer's preference for communicating in a particular format C.The details about the audience that may be unknown to the writer D.The writer's ability to address the audience directly

A.The ways in which the audience expects to receive communications Option (A) is correct. This question tests your ability identify the impact of audience on a particular writing task. The intended audience for this piece of writing is public and school officials, who expect to receive communications from the public through letters or emails.

Which THREE of the following are appropriate purposes for student journals? Select all that apply. A.To provide a place for students to articulate their thoughts about course material B.To serve as a means of evaluating a student's facility with mechanics and rhetorical strategies C.To provide a place for students to compose final drafts of their writing D.To help students find personal connections to the material they are studying E.To provide students with a place to make predictions about what they are reading

A.To provide a place for students to articulate their thoughts about course material D.To help students find personal connections to the material they are studying E.To provide students with a place to make predictions about what they are reading Options (A), (D), and (E) are correct. This question tests your knowledge of approaches to teaching writing. By its very nature, a journal is an informal record of thoughts and experiences and is generally kept for private use. The keeping of journals is a way of encouraging students to write, and write often, on a wide variety of matters that concern them. Because of the informal nature of such journals, students may be less likely to observe the formal rules of mechanics or to use rhetorical strategies demanded in more formal writing; therefore, evaluating students in this way would be inappropriate.

(1) People have debated the best ways to educate children for decades. (2) One of the many aspects of education that is often debated is the structure of the school years. (3) Historically, students were given summers off so they could help their families harvest crops. (4) As more families got away from farming, this became less necessary, but many argued that money is saved by closing schools for the summer. (5) With other countries surpassing the test scores of American students, the concept of year-round schooling is again being pushed by lawmakers. Which of the following errors is present in sentence 4 ? A.Unclear pronoun reference B.Incorrect subject-verb agreement C.Misplaced modifier D.Faulty comma placement

A.Unclear pronoun reference Option (A) is correct. This question tests your ability to recognize errors in standard written English. In the sentence, the pronoun "this" has an unclear referent.

It took well over a year. I don't mean for us. I mean for the animals. Considering that animals dispense with clothes, footwear, linen, furniture, kitchenware, toiletries; that nationality means nothing to them; that they care not a jot for passports, money, employment prospects, schools, cost of housing, healthcare facilities—considering, in short, their lightness of being, it's amazing how hard it is to move them. Moving a zoo is like moving a city. —Yann Martel, The Life of Pi Question:The subject of the final sentence in the passage is an example of A.a gerund phrase B.a participial phrase C.a prepositional phrase D.a noun phrase

A.a gerund phrase Option (A) is correct. This question tests your knowledge of grammatical elements in a sentence. The subject of the sentence is the phrase "Moving a zoo." "Moving" is a gerund, which is a verb ending with "-ing" that is used as a noun. Therefore, the subject is a gerund phrase and (A) is the best answer.

Dr. Johnson was observed by a musical friend of his to be extremely inattentive at a concert while a celebrated soloist played virtuosic flurries of notes on his violin. His friend, to induce him to take greater notice of what was going on, told him how extremely difficult it was to follow the performance. "Difficult do you call it, sir?" replied the doctor. "I wish it were impossible." Question:The tone of Johnson's statement suggests that he was A.annoyed by the performance of the solo player B.irate at the interruption by his friend C.discouraged by his friend's inability to understand the music D.disheartened by the solo player's egotistical attitude

A.annoyed by the performance of the solo player Option (A) is correct. This question asks you to identify what was annoying Dr. Johnson. Johnson's friend comments that the musical performance was "difficult" to follow. He is using "difficult" in one of its senses to mean "hard to understand." Johnson replies by using a different sense of "difficult," meaning "hard to deal with, manage, or overcome." By using this second meaning of "difficult," Johnson is indicating that he is annoyed by the performance itself.

In a drama, the main character sustains the action in the plot, while the ___________ serves as the opposing element throughout the plot. A.antagonist B.chorus C.ingenue D.persona

A.antagonist Option (A) is correct. This question tests your ability to identify typical characteristics of a genre. The sentence provides a partial description of the functions of the main character, or protagonist, and the opposing character, or antagonist, in a literary drama.

The term regionalism, once honorably used in the United States to identify the literature of a young and far-flung nation, now too often suggests narrowness and parochialism, a mentality afflicting authors too timid to make it in the big city. Such _________ are, of course, unfair; a novel set in Manhattan's East Side, for example, can be many times more provincial than a tale from the hills or the hollows. Question:Which of the following correctly fills the blank? A.connotations B.epigrams C.euphemisms D.denotations

A.connotations Option (A) is correct. This question tests your knowledge of semantics, specifically the ability to recognize a reference to connotation. The passage states that "narrowness" and "parochialism" are qualities that have become associated with the term "regionalism." Connotations are notions suggested by or associated with a word that go beyond the word's explicit, or denoted meaning.

(1) How does the work of the journalist differ from that of the historian? (2) The most obvious difference is that the journalist reports the everyday triumphs and follies of the human race and has little time to reflect on these occurrences, whereas the historian must distinguish between what is trivial and what is important in both the past and the present. (3) As a journalist, I report as news today stories that may or may not be treated as history in later generations. (4) The historian will sort through masses of information and create what it is hoped will be a graceful accounting of what happened, why, and with what result. Question:Taken together, the tone, language, and content of the passage indicate that it was most likely written to A.convince a general readership of the complementary functions of the work of journalists and historians B.suggest to historians that a journalistic approach could be a useful approach for historians to take C.create interest in the field of journalism for individuals deciding on a subject of study D.make the case to a general audience that journalists are just as important as historians are

A.convince a general readership of the complementary functions of the work of journalists and historians Option (A) is correct. The writer begins the passage by explaining the difference between journalists and historians: journalists write about the everyday events and historians distinguish between the trivial and the important in the past and present. The writer then shows how the work of both professions is connected or complementary. Journalists report the news of today that the historians will sort through, determining first what is important, and then writing an account of what happened, why, and with what result.

My mistress' eyes are nothing like the sun; Coral is far more red than her lips' red; If snow be white, why then her breasts are dun; If hairs be wires, black wires grow on her head. Question:In these lines, the speaker A.departs from the Renaissance sonnet-writer's stock conventions of praise B.likens the battle of love, through imagery of red and white, to the War of the Roses C.uses traditional Renaissance imagery of color and color contrasts to communicate a new perspective on art D.plays against the rhythms of iambic pentameter in each of the four lines

A.departs from the Renaissance sonnet-writer's stock conventions of praise Option (A) is correct. This question tests your knowledge of the conventions of a style. The conventional Renaissance (or Petrarchan) sonnet writer glorified a fair-skinned woman with red lips, sparkling eyes, white breasts, and golden hair. Here, the speaker asserts at the start of the sonnet that his lover has no such beauty or graces. His straightforward announcement of what she lacks rather than of what she has is unexpected and puts the reader off guard. This unconventional approach leads to a resolution in the final couplet of the sonnet, where the speaker asserts that, even without the conventional marks of "fairness," his love is more beautiful than any woman "belied with false compare."

My sister, Mrs. Joe Gargery, was more than twenty years older than I, and had established a great reputation with herself and her neighbours because she had brought me up "by hand." Having at that time to find out for myself what the expression meant, and knowing her to have a hard and heavy hand, and to be much in the habit of laying it upon her husband as well as upon me, I supposed that Joe Gargery and I were both brought up by hand. She was not a good-looking woman, my sister, and I had a general impression that she must have made Joe Gargery marry her by hand. Joe was a fair man, with curls of flaxen hair on each side of his smooth face, and with eyes of such a very undecided blue that they seemed to have somehow got mixed with their own whites. He was a mild, good-natured, sweet-tempered, easy-going, foolish, dear fellow—a sort of Hercules in strength and also in weakness. —Charles Dickens, from Great Expectations Question:The point of view in the passage is A.first person B.second person C.third person, limited D.third person, omniscient

A.first person Option (A) is correct. This question tests your knowledge of narrative point of view. The narrator is a character in the work; uses the first person pronoun ("I"); and limits the narrative to personal experience, conjecture, and opinion. Therefore, he is a first-person narrator and (A) is the correct answer.

Just then something long, round, limp and black fell upon her shoulders and slithered to the floor beside her. A great terror took hold of her. It softened her knees and dried her mouth so that it was a full minute before she could cry out or move. Then she saw that it was the big bull whip her husband liked to carry when he drove. She lifted her eyes to the door and saw him standing there bent over with laughter at her fright. She screamed at him. "Sykes, what you throw dat whip on me like dat? You know it would skeer me—looks just like a snake, an' you knows how skeered Ah is of snakes." —Zora Neale Hurston, from the short story "Sweat"(1926) Zora Neale Hurston often had her characters speak using African American dialect to A.highlight the richness of the authentic oral language and culture of African Americans B.share with the world the reality of life for African Americans in the Deep South C.show how spoken language varied among African Americans in different socioeconomic groups D.protest the use of informal language in narratives that depict African Americans

A.highlight the richness of the authentic oral language and culture of African Americans Option (A) is correct. This question tests your ability to recognize how dialect is used in literature. Hurston is known for her commitment to capturing the sound of the speech of African Americans in her writing. In her own time, she was criticized for maintaining stereotypes of African American speech, but she later inspired other writers like Alice Walker to use authentic Black dialect in writing.

When the English tongue we speak, Why is break not rhymed with freak? Will you tell me why it's true We say sew, but likewise few? And the maker of a verse Cannot cap his horse with worse? Question:The lines above deal with English A.orthography B.metrics C.dialects D.semantics

A.orthography Option (A) is correct. This question tests your knowledge of orthography and the relationship between spelling and pronunciation. The lines make the point that often the spellings, or orthography, of words in English do not provide consistent guides to pronunciation. Rhymed words share the same last stressed vowel and the same speech sounds following that vowel. But words sharing identical spellings of those elements do not necessarily rhyme, as the examples of "break" and "freak" and "horse" and "worse" demonstrate. Conversely, though one would not expect words spelled with different vowels to rhyme, the pair "verse" and "worse" shows that sometimes they do.

The passage below is taken from N. Scott Momaday's The Way to Rainy Mountain, a collection of Kiowa legends. A single knoll rises out of the plain in Oklahoma, north and west of the Wichita Range. For my people, the Kiowas, it is an old landmark, and they gave it the name Rainy Mountain. The hardest weather in the world is there. Winter brings blizzards, hot tornadic winds arise in the spring, and in summer the prairie is an anvil's edge. The grass turns brittle and brown, and it cracks beneath your feet. There are green belts along the rivers and creeks, linear groves of hickory and pecan, willow and witch hazel. At a distance in July or August the steaming foliage seems almost to writhe in fire. Great green and yellow grasshoppers are everywhere in the tall grass, popping up like corn to sting the flesh, and tortoises crawl about on the red earth, going nowhere in the plenty of time. Loneliness is an aspect of the land. All things in the plain are isolate; there is no confusion of objects in the eye, but one hill or one tree or one man. To look upon that landscape in the early morning, with the sun at your back, is to lose the sense of proportion. The tone of the passage is best described as A.reverent B.ironic C.scornful D.jovial

A.reverent Option (A) is correct. The question tests your ability to identify the tone of a passage. While a number of words could be used to describe the tone of this passage, the most appropriate of the choices given is "reverent." The author is painting a picture of a barren, stark, and challenging landscape. He describes it as lonely, isolating, but awe-inspiring. Most important is the reference, "For my people, the Kiowas, it is an old landmark. . . ."

_________. A wreck might be valuable for itself, of course, rather than for its cargo or the scrap value of its hull and fittings. A ship like the Mary Rose is of historical interest, and her salvaging was more in the nature of an archaeological dig that happened not to be performed on dry land. The Titanic, too, is now viewed in a similar light, thanks mainly to some very well orchestrated publicity intended to supplant the ship's poetic and melancholy status with that of a treasure trove of artifacts. References to the Mary Rose and the Titanic in the paragraph provide A.supporting details B.necessary transitions C.a cause-and-effect relationship D.appropriate generalizations

A.supporting details Option (A) is correct. This question tests your ability to recognize the function of certain references. Discussions of the two specific ships supply supporting examples for the assertion in the second sentence that "A wreck might be valuable for itself..."

So your customers have ordered a lot of your widgets online. Congratulations! But don't start counting your chickens: You still have some work to do before you make those sales. You can't send customers their products until you calculate taxes and shipping. And more importantly, you still need to determine how to process the customers' preferred methods of payment. The key is to figure out the best software solution for your situation. Which of the following does the author do to gain the attention of the reader? A.Uses euphemism B.Addresses the reader directly C.Predicts future profits D.Attacks a popular idea

B.Addresses the reader directly Option (B) is correct. This question asks you to identify a technique used by the author to gain the attention of the reader. Throughout the passage the author addresses the reader directly: "your customers," "you still need," and "You can't send. . . ."

During the Second World War, Japanese Americans who lived in the United States were sent to internment camps until the war was over. In her memoir Farewell to Manzanar, Jeanne Wakatsuki tells the story of her internment at one of the camps, Manzanar. The excerpt below describes her first observations upon arriving at the camp with her family at age seven. (1) I could see a few tents set up, the first rows of black barracks, and beyond them, blurred by sand, rows of barracks that seemed to spread for miles across this plain. (2) People were sitting on cartons or milling around, with their backs to the wind, waiting, to see which friends or relatives might be on this bus. (3) As we approached, they turned or stood up, and some moved toward us expectantly. (4) But inside the bus no one stirred. (5) No one waved or spoke. (6) They just stared out the windows, ominously silent. (7) I didn't understand this. (8) Hadn't we finally arrived, our whole family intact? (9) I opened a window, leaned out, and yelled happily, "Hey! This whole bus is full of Wakatsukis!" Which of the following assessment tools would provide the most detailed feedback for students giving oral presentations about their inquiry projects? A.A holistic rubric B.An analytic rubric C.A primary trait scoring rubric D.A self-assessment rubric

B.An analytic rubric Option (B) is correct. This question tests your ability to recognize a variety of approaches for summative assessment of an oral presentation. While all of the options are assessment tools, only an analytic rubric lists critical dimensions of performance (e.g., eye contact, voice modulation) and then describes the level of proficiency for each dimension (e.g., novice, apprentice, proficient, distinguished). The analytic rubric would provide the most detailed feedback to students.

Parents who decide to school their children at home commit time and energy to an activity that once was left to specialized professionals. Even in the states with the most permissive home-schooling laws, parents must learn what is normally taught to children of a given age, find materials and projects that teach specific skills, and learn how to use their own time and that of their children productively. The vast majority of home-schooling parents who hope their children will attend college also must learn how to assess their children's progress against higher-education admission standards. Question:The excerpt above is most likely taken from which of the following? A.A textbook on the history of education B.An instructional manual on home schooling C.A journal article for new teachers D.An admissions brochure for a university

B.An instructional manual on home schooling Option (B) is correct. The tone of the passage is informative, providing instructions concerning home-schooling. The passage includes information about what is required for parents to do, such as learning appropriate curriculum, finding materials, managing time, and assessing children's progress to assure that they meet higher education standards. Therefore, the passage serves as an instruction manual for home-schooling.

Which of the following novels focuses primarily on the concept of individualism versus collectivism? A.Things Fall Apart by Chinua Achebe B.Anthem by Ayn Rand C.Lord of the Flies by William Golding D.The Bluest Eye by Toni Morrison

B.Anthem by Ayn Rand Option (B) is correct. This item tests your knowledge of the literary contexts of major works. Anthem deals mainly with the main character's struggle to break free of his collectivist society and become an individual.

After hearing his students discuss what a tragedy might be, a teacher observes that they have misconceptions that may interfere with their understanding of a tragedy they will be reading. He prepares a list of statements about what constitutes a tragedy, and asks students to indicate whether they agree or disagree with each statement. After reading the play, the class will discuss what their misconceptions were and how they have revised their thinking. Question:Which of the following comprehension strategies did the teacher use? A.Semantic feature analysis B.Anticipation guide C.Reciprocal teaching D.Background building

B.Anticipation guide Option (B) is correct. This question tests your knowledge of important techniques for aiding comprehension. An anticipation guide is a series of questions that students are asked to respond to (usually by marking "Agree" or "Disagree") before a particular unit or lesson is begun. After the unit or lesson, the students review their answers to the anticipation guide and reflect on what they know or understand better.

Over the past few decades, the health of American students has greatly declined. Young adults are increasingly being diagnosed with health problems that used to be more typical of people over 50, such as blood pressure issues, type 2 diabetes, and obesity. Experts point to changes in diet and declining levels of physical activity as contributing factors. Since young people spend so much of their time at schools, many experts believe schools have an obligation to address this issue. Propose a course of action for your school to take in order to address this health issue. Explain why the school should address the issue and why the school should choose your course of action over other possible solutions. Be sure that you address your writing to the appropriate audience that has the power to make decisions about the issue. Which mode of writing is most appropriate for the task presented in the prompt? A.Narrative B.Argumentative C.Expository D.Descriptive

B.Argumentative Option (B) is correct. This question tests your ability to recognize the appropriateness of different modes of writing. The prompt asks students to take a position, propose a course of action, and explain why their solution is the best choice, thereby persuading readers to agree with their proposal. Argumentative writing includes the ability to persuade.

After the school board announced that it would be ending instrumental music programs in the school, the high school's English teachers asked their students to share their reactions to the announcement with the editor of the school newspaper. A tenth-grade student submitted the following response. Dear Editor, (1) I have been playing the violin for six years. (2) Music is an important part of how I express myself. (3) Seeking instruction outside of school is not an option for me. (4) If the music program is cut, I will not be able to continue my lessons. (5) I don't think that I am the only person who is in this position. (6) All of my friends in orchestra are sad to see the instrumental programs go. Anna Question:Which of the following transitions is most appropriate to insert at the beginning of sentence 4 ? A.However, B.Consequently, C.At any rate, D.Rather,

B.Consequently, Option (B) is correct. This question tests your facility with writing conventions. "Consequently," indicates a cause-and-effect relationship. In the response, the student will have to give up music lessons as a result of her not being able to seek instruction outside of school. None of the other transitions indicates a correct relationship between the two ideas.

(1) People have debated the best ways to educate children for decades. (2) One of the many aspects of education that is often debated is the structure of the school years. (3) Historically, students were given summers off so they could help their families harvest crops. (4) As more families got away from farming, this became less necessary, but many argued that money is saved by closing schools for the summer. (5) With other countries surpassing the test scores of American students, the concept of year-round schooling is again being pushed by lawmakers. A style manual will best help the student in which of the following areas of writing? A.Varying word choice B.Documenting sources C.Providing background information D.Strengthening point of view

B.Documenting sources Option (B) is correct. This question tests your knowledge of reference materials. A style manual provides information about writing and formatting for a variety of purposes. A style manual would include guidelines for incorporating informational interviews into an essay.

A. For thou hast sent her a mantle of green, As green as any grass, And bade her come to the silver wood To hunt with Child Maurice. B. An Ace of Hearts steps forth: the King unseen Lurked in her hand, and mourned his captive Queen: He springs to vengeance with an eager pace, And falls like thunder on the prostrate Ace. The nymph exulting fills with shouts the sky; The walls, the woods, and long canals reply. C. Who would have thought my shriveled heart Could have recovered greenness? It was gone Quite underground; as flowers depart To see their mother-root, when they have blown, Were they together All the hard weather, Dead to the world, keep house unknown. D. But wherefore rough, why cold and ill at ease? Aha, that is a question! Ask, for that, What knows,—the something over Setebos That made Him, or He, may be, found and fought, Worsted, drove off and did to nothing, perchance. Question:Which is from a mock epic? A.Excerpt A B.Excerpt B C.Excerpt C D.Excerpt D

B.Excerpt B Option (B) is correct. This question tests your knowledge of literary genres. Mock epics adopt the elevated style of the epic — including such epic conventions as invocations, formal diction, extended similes, lengthy descriptions of battles, and supernatural interventions — to treat trivial subjects. The disjunction between style and subject usually achieves a satirical effect. The lines in (B), from Alexander Pope's mock epic, "The Rape of the Lock," present a card game as though it were a battle; in winning the game, Pope's heroine Belinda (referred to as a "nymph" in the fifth line) wins a round in the battle of the sexes that is satirized throughout the poem.

Question:Which of the following terms best describes a type of fiction that makes use of the grotesque, violent, mysterious, and supernatural? A.Realistic B.Gothic C.Modernist D.Absurdist

B.Gothic Option (B) is correct. This question asks you to identify a type of fiction based on its definition. Gothic fiction is the only choice that matches the description in the question. Well-known examples of Gothic fiction include Mary Shelley's Frankenstein and Horace Walpole's The Castle of Otranto.

After the school board announced that it would be ending instrumental music programs in the school, the high school's English teachers asked their students to share their reactions to the announcement with the editor of the school newspaper. Which of the following strategies is best for an English teacher to use when guiding students to select the most appropriate mode of writing for the assignment? A.Holding a class discussion about arts in education B.Having students brainstorm a variety of situations in which they have been asked to express their opinions C.Dividing the class into groups and providing the students with newspapers to use for inspiration D.Modeling an appropriately written letter and having students copy its structure

B.Having students brainstorm a variety of situations in which they have been asked to express their opinions Option (B) is correct. This question tests your ability to recognize how mode is related to a writer's purpose. Brainstorming situations in which students have been asked to provide an opinion will establish purpose and direction for writing, and will therefore support students in choosing the most appropriate mode of writing for the assignment.

A high school ELA teacher creates the checklist below based on commonly observed weaknesses in student writing. Writers Check List __ Sentences begin in different ways. __ Sentences build on the ones that precede them. __ Sentences vary in length. __ The meaning of each sentence is clear. __ Sentences flow and are grammatically correct. __ There are no run-on sentences. __ Sentences are complete. Question:How can the teacher most effectively use the checklist to assess student mastery of the items in a final draft? A.Using checklist items as the basis for a multiple-choice exam B.Including the checklist items in a rubric for an original student essay C.Instructing students to use the checklist during peer review discussions D.Having students use the checklist to critique a teacher-provided passage

B.Including the checklist items in a rubric for an original student essay Option (B) is correct. This question tests your ability to recognize purposes and methods of assessment. Including the checklist items in a rubric used to grade a student essay allows the teacher to assess whether the student has made progress in mastering the writing conventions addressed in the checklist. Although using the checklist during a peer review discussion would be an effective use of this tool, it would not allow the teacher to effectively assess the student's mastery on a final draft.

Prefix: not Root: believe Suffix: possible to be Question:Which of the following words can be defined based on the word roots listed above? A.Incredulous B.Incredible C.Discredit D.Discreditably

B.Incredible Option (B) is correct. This question tests your knowledge of affixes. The root word "cred," which means believe, is used in all options. The prefix "in" means not. The suffix "-ible" means possible to be. The suffix "-ulous" means inclined to. Therefore,"incredible" can be broken down into its parts to mean "not possible to be believed."

The parking lot is open to members only from 5 A.M. to 7 P.M. Question:Which of the following will resolve the ambiguity of the sentence above? A.Insert the word large before parking lot B.Move only from after members to after 7 P.M. C.Change The parking lot is to These parking lots are D.Change is open to opens

B.Move only from after members to after 7 P.M. Option (B) is correct. This question tests your ability to recognize how meaning is affected by word order. The ambiguity of the original sentence results from the placement of the modifier "only"; a reader cannot tell whether "only" is intended to modify "members" or the phrase "from 5 A.M. to 7 P.M." If "only" refers to "members," the sentence means that nonmembers cannot park in the parking lot between 5 A.M. and 7 P.M., but members can. If "only" refers to the time period between 5 A.M. and 7 P.M., however, the meaning of the sentence is somewhat different; it means that members can park in the lot from 5 A.M. to 7 P.M., but cannot park there at other times. By changing the position of "only" to make clear that it modifies "from 5 A.M. to 7 P.M.," (B) resolves the ambiguity of the sentence and is, therefore, the best answer.

Just then something long, round, limp and black fell upon her shoulders and slithered to the floor beside her. A great terror took hold of her. It softened her knees and dried her mouth so that it was a full minute before she could cry out or move. Then she saw that it was the big bull whip her husband liked to carry when he drove. She lifted her eyes to the door and saw him standing there bent over with laughter at her fright. She screamed at him. "Sykes, what you throw dat whip on me like dat? You know it would skeer me—looks just like a snake, an' you knows how skeered Ah is of snakes." —Zora Neale Hurston, from the short story "Sweat"(1926) "A great terror took hold of her. It softened her knees and dried her mouth so that it was a full minute before she could cry out or move." Question:Which of the following literary devices is used in the sentence to describe the character's feelings? A.Metaphor B.Personification C.Alliteration D.Assonance

B.Personification Option (B) is correct. This question tests your ability to analyze figures of speech. Personification is the attribution of human characteristics to nonhuman things. The "terror" is described as "[taking] hold of her." Because terror is a feeling, it cannot literally hold her. The correct answer, therefore, is (B).

The following excerpt is from William Shakespeare's King Lear. See how yond justice rails upon yond simple thief.Hark in thine ear. Change places and handy-dandy,Which is the justice, which is the thief? Through tatter'd clothes small vices do appear.Robes and furr'd gown hide all. Plate sin with gold,And the strong lance of justice hurtless breaks;Arm it in rags, a pygmy's straw does pierce it. Question:Which of the following is the best summary of this passage? A.Everyone deserves a chance to reform. B.Riches and position hide guilt. C.No one can escape retribution. D.Justice is blind.

B.Riches and position hide guilt. Option (B) is correct. This question tests your ability to interpret the meaning of the dramatic speech. The passage suggests that the poor and powerless have no way of concealing their vices, while the "Robes and furr'd gowns" worn by people of influence, particularly judges and members of the nobility, cover the guilt of the wearers. The shield metaphor in the last three lines conveys a similar meaning: gold, or money, effectively conceals and protects the sinner from the "lance of justice," or the force of law.

A hush is over all the teeming lists, And there is pause, a breath-space in the strife; A spirit brave has passed beyond the mists And vapors that obscure the sun of life. And Ethiopia, with bosom torn, Laments the passing of her noblest born. For her his voice, a fearless clarion, rung That broke in warning on the ears of men; For her the strong bow of his power he strung, And sent his arrows to the very den Where grim Oppression held his bloody place And gloated o'er the mis'ries of a race. Through good and ill report he cleaved his way Right onward, with his face set toward the heights, Nor feared to face the foeman's dread array,— The lash of scorn, the sting of petty spites. He dared the lightning in the lightning's track, And answered thunder with his thunder back. —Paul Laurence Dunbar, from "Frederick Douglass" A. Student A: This poem is about war. It's about "strife" or "bloody" fighting. The champion "cleaved his way Right onward." B. Student B: A famous leader is praised for all the contributions he made to his people. He met much oppression in his struggle for his people but overcame it all. C. Student C: This lady, Ethiopia, is very sad when someone close to her dies. This person, who was killed in war, helped people. He must have been some kind of a super warrior or maybe even a god. D. Student D: It interested me the way this poem is so concerned with sound and noise. It begins on a quiet note and death but then builds up to the final stanza which is full of sound words and action. Which student response provides the best summary of the poem? A.Student A B.Student B C.Student C D.Student D

B.Student B Option (B) is correct. This question tests your ability to interpret a poem and recognize an appropriate summary. The poem's subject, as indicated by the title, is Frederick Douglass, an escaped slave who spent most of his life fighting for the rights of African Americans and members of other oppressed groups. It is not, however, necessary to be familiar with Douglass' life to interpret and summarize the poem. As student B notes, the subject is clearly a brave leader who overcame his struggles. The first sentence of student B's response summarizes the first two stanzas of the poem; the second sentence summarizes the third stanza.

Metaphoric thinking is a natural part of language. In school, when children are struggling to explain an idea, they may say "It's something like . . . " and we sense, in Joan Solomon's words, that "another domain of experience is being used in an imaginary way." This is certainly true in science and math where similes and metaphors underlie the mental modeling that is crucial for learning and understanding these subjects. An example of this is when students are taught to think of addition as being similar to piling up blocks. Question:Which of the following is most similar to the example provided in the last sentence of the passage? A.Students are taught to recognize bird species by the variations in their colors. B.Students are taught to think of electricity as being analogous to the flow of water. C.Students are shown pictures of orchids and taught to appreciate their beauty. D.Students learn about the circulatory system through a series of diagrams.

B.Students are taught to think of electricity as being analogous to the flow of water. Option (B) is correct. The passage describes the use of metaphoric language, particularly similes and metaphors, to explain concepts. The last sentence compares the action of piling blocks to addition. In (B), electricity is described as being similar (or analogous) to the action of flowing water.

Which of the following is the best example of a kinesthetic activity that benefits English-language learners who are learning new vocabulary words? A.Students join cooperative groups to discuss the words and their meanings. B.Students draw pictures to define the vocabulary words. C.Students read the words aloud from a list. D.Students record the pronunciation of the words.

B.Students draw pictures to define the vocabulary words. Option (B) is correct. This question tests the ability to recognize a kinesthetic activity as it applies to vocabulary acquisition. (A), (C), and (D) are auditory activities. Drawing pictures is a kinesthetic activity because the student is interacting with the word by creating a visual representation of it.

The Secret Life of Bees was written by which of the following authors? A.Margaret Mitchell B.Sue Monk Kidd C.Bailey White D.Alice Walker

B.Sue Monk Kidd Option (B) is correct. Sue Monk Kidd is the author of The Secret Life of Bees.

Which of the following novels are paired with their corresponding author? Select all that apply. A.Beloved - Alice Walker B.The Joy Luck Club - Amy Tan C.The Color Purple - Maya Angelou D.The Woman Warrior - Maxine Hong Kingston E.Their Eyes Were Watching God - Zora Neale Hurston

B.The Joy Luck Club - Amy Tan D.The Woman Warrior - Maxine Hong Kingston E.Their Eyes Were Watching God - Zora Neale Hurston Options (B), (D), and (E) are correct. Amy Tan is the author of The Joy Luck Club, Alice Walker is the author of The Color Purple, Maxine Hong Kingston is the author of The Woman Warrior, and Zora Neale Hurston is the author of Their Eyes Were Watching God. Beloved was written by Toni Morrison, not Alice Walker, who wrote The Color Purple.

Students have written a creative response to a story they have read. Which of the following might be an effective strategy for assessing what they have written? A.The teacher asks each student to develop guidelines after completing the assignment and then the teacher assesses each paper based on the guidelines. B.The teacher works together with the students ahead of time to develop a scoring guideline so that they know the expectations for the assignment. C.The teacher asks students to read and respond to each others' papers using a scoring guideline that the teacher developed after the assignment was given. D.The teacher asks the students to develop guidelines before completing the assignment and then the teacher assesses each paper using a rubric that the teacher developed.

B.The teacher works together with the students ahead of time to develop a scoring guideline so that they know the expectations for the assignment. Option (B) is correct. This question tests your ability to recognize appropriate assessment techniques for writing. Students working on any writing assignment do best when they know the expectations and the audience prior to completing the assignment. While the activity in (D) does include students in the development of assignment criteria, it does not apply that criteria to the assessment.

Aristotle moved from place to place in the Lyceum while he taught. He was the prototype of peripatetic teachers. So say those who enjoyed not only hours, but days, walking with C. S. Lewis or Francis Schaeffer. Question:To determine the meaning of the underlined word in the passage above, a student would find which of the following most helpful? A.The use of structural cues B.The use of context cues C.Understanding of figurative language D.Understanding of euphemisms

B.The use of context cues Option (B) is correct. This question tests your knowledge of strategies for building students' vocabulary. The word peripatetic is not itself, nor is it surrounded by, figurative language or euphemisms. It does not come from a common Latin or Greek root, nor does it use a common prefix or suffix that would lend help from structural cues. The context around the word ("moved from place to place," "walking with C. S. Lewis or Francis Schaeffer") gives enough clues to help a student figure out that the word must mean "walking about."

The passage below is taken from N. Scott Momaday's The Way to Rainy Mountain, a collection of Kiowa legends. A single knoll rises out of the plain in Oklahoma, north and west of the Wichita Range. For my people, the Kiowas, it is an old landmark, and they gave it the name Rainy Mountain. The hardest weather in the world is there. Winter brings blizzards, hot tornadic winds arise in the spring, and in summer the prairie is an anvil's edge. The grass turns brittle and brown, and it cracks beneath your feet. There are green belts along the rivers and creeks, linear groves of hickory and pecan, willow and witch hazel. At a distance in July or August the steaming foliage seems almost to writhe in fire. Great green and yellow grasshoppers are everywhere in the tall grass, popping up like corn to sting the flesh, and tortoises crawl about on the red earth, going nowhere in the plenty of time. Loneliness is an aspect of the land. All things in the plain are isolate; there is no confusion of objects in the eye, but one hill or one tree or one man. To look upon that landscape in the early morning, with the sun at your back, is to lose the sense of proportion. The passage suggests which of the following about the Kiowas? A.They have created monuments on Rainy Mountain. B.They are a very tough, resilient people. C.They visit but do not live on Rainy Mountain. D.They include both farmers and hunters.

B.They are a very tough, resilient people. Option (B) is correct. This question tests your ability to draw an inference from informational material. The passage describes the landscape in which the Kiowa people live. It is a very harsh, sometimes dangerous, and challenging environment. In order to survive here, one can infer that the Kiowa people are tough and resilient.

Simplistic is often used incorrectly because of its resemblance to the word simple. In which of the following sentences is the word simplistic used correctly? Select all that apply. A.Your simplistic ideas sound sensible to me. B.Your simplistic solutions do not address the difficult situation. C.How shall we assess the student's simplistic yet heartfelt response to poetry? D.To keep things simplistic, we suggest you copy and paste the text into the document. E.Everything fell in to place; it was fairly simplistic.

B.Your simplistic solutions do not address the difficult situation. C.How shall we assess the student's simplistic yet heartfelt response to poetry? Options (B) and (C) are correct. This question tests your ability to detect correct uses of a word that is often used incorrectly. Simplistic carries a negative connotation, implying a simple-minded or insufficiently complex approach to an issue.

The previously unpublished writers in this anthology are part of a recent phenomenon—in their numbers probably a unique phenomenon in history. They are not an organized movement; they don't have a common politics; they don't even have a name. Are they "new/old" writers or "promising old" writers—that juxtaposition of adjectives is peculiar to the ears; are they "late bloomers" or simply late-developing writers? —Sondra Zeidenstein, from A Wider Giving: Women Writing after a Long Silence The passage is primarily concerned with A.discussing the work of a particular group of writers B.attempting to characterize an unusual group of writers C.expressing doubt about the quality of certain writers' work D.comparing the accomplishments of several different writers

B.attempting to characterize an unusual group of writers Option (B) is correct. The passage attempts to find a category in which to put these previously unpublished authors.

The passage below is taken from N. Scott Momaday's The Way to Rainy Mountain, a collection of Kiowa legends. A single knoll rises out of the plain in Oklahoma, north and west of the Wichita Range. For my people, the Kiowas, it is an old landmark, and they gave it the name Rainy Mountain. The hardest weather in the world is there. Winter brings blizzards, hot tornadic winds arise in the spring, and in summer the prairie is an anvil's edge. The grass turns brittle and brown, and it cracks beneath your feet. There are green belts along the rivers and creeks, linear groves of hickory and pecan, willow and witch hazel. At a distance in July or August the steaming foliage seems almost to writhe in fire. Great green and yellow grasshoppers are everywhere in the tall grass, popping up like corn to sting the flesh, and tortoises crawl about on the red earth, going nowhere in the plenty of time. Loneliness is an aspect of the land. All things in the plain are isolate; there is no confusion of objects in the eye, but one hill or one tree or one man. To look upon that landscape in the early morning, with the sun at your back, is to lose the sense of proportion. "Winter brings blizzards, hot tornadic winds arise in the spring, and in summer the prairie is an anvil's edge." Question:This sentence is an example of a A.simple sentence B.compound sentence C.complex sentence D.compound-complex sentence

B.compound sentence Option (B) is correct. This question tests your ability to identify sentence structures. Compound sentences consist of two or more independent clauses.

In preparation for a class discussion, students in a high school English Language Arts class work in pairs to formulate an opinion about the discussion topic. After working in pairs, the students then share their opinions with the class. The technique described best supports students' productive participation in the class discussion by A.requiring different perspectives B.encouraging individual accountability C.providing engaging discussion topics D.modeling acceptable discussion behavior

B.encouraging individual accountability Option (B) is correct. This question tests your ability to recognize strategies for encouraging productive participation in a class discussion. Collaborating in pairs before sharing with the class encourages each student to contribute to the discussion in a productive way. Allowing students to form an opinion with a partner before discussing it with the entire class holds each student accountable for considering the discussion topic.

So your customers have ordered a lot of your widgets online. Congratulations! But don't start counting your chickens: You still have some work to do before you make those sales. You can't send customers their products until you calculate taxes and shipping. And more importantly, you still need to determine how to process the customers' preferred methods of payment. The key is to figure out the best software solution for your situation. Question:The excerpt appeals to its target audience by A.comparing the target audience's product to another product B.praising the success of the target audience C.portraying the interests of the target audience's customers in a positive way D.suggesting that the members of the target audience are highly respected in society

B.praising the success of the target audience Option (B) is correct. This question asks you to identify how the advertisement appeals to its target audience. The advertisement congratulates the target audience on its orders before proceeding to a warning about collecting payment and calculating net profit.

Because mistakes of all kind are so common, we tend to have a soft spot for errors and those who make them. The first comedy album to sell a million records was Radio Bloopers, issued in 1954. We have made best sellers out of such books as Murphy's Law, a compendium of sayings that touch on the general tendency both to make mistakes and to marvel at the foul-ups of others. Question:The author of the passage supports the statement made in the first sentence by A.citing personal experience B.presenting supporting examples C.appealing to the reader's emotions D.pointing out an incorrect assumption

B.presenting supporting examples Option (B) is correct. The first sentence provides the main idea of the passage: because mistakes are so common, we are accepting and fond of errors and those who make them. The rest of the paragraph provides examples of humorous collections of errors, such as a comedy album of bloopers and best-selling books that focus on the mistakes people make.

Code switching refers to A.the ability of bilingual individuals to think in one language while speaking in another B.the ability to modify one's language according to audience and purpose C.the ability of mothers to communicate with their infants D.the ability of computers to generate and change secure forms of communication on demand

B.the ability to modify one's language according to audience and purpose Option (B) is correct. This questions tests your ability to recognize terms associated with dialect and diction. Code-switching is a well-documented linguistic move that is defined as the ability to modify one's language according to audience and purpose. It is particularly associated with speakers of a dialect of English (e.g., African American English) as their home language.

Which of the following is an example of a formative assessment of a writing assignment? A. Grouping weaker writers with stronger writers for peer reviews B. Using a predetermined rubric to grade the writing assignment C. Holding individual writing conferences with students to discuss their drafts D. Giving point values to drafts at each stage of the writing process

C. Holding individual writing conferences with students to discuss their drafts Option (C) is correct. This question tests your ability to understand the purposes and methods of assessing writing. Formative assessments are intended to help the teacher modify instruction and gauge student understanding throughout the learning process. Conferencing about a draft is a formative assessment because conferences help students revise their work before submitting a final product and help the teacher assess student understanding and modify instruction if needed.

Which of the following statements is supported by the writing across the curriculum movement? A. Grammar drills lead to overall improvement of student writing. B. Students develop as writers by learning the five-paragraph essay structure before writing lengthier essays. C. Students benefit from writing in a variety of forms in a variety of situations. D. Students should be provided with specific topics for all writing assignments.

C. Students benefit from writing in a variety of forms in a variety of situations. Option (C) is correct. This question tests your ability to recognize research-based strategies for writing instruction. Writing across the curriculum is a pedagogical movement with a foundation in the importance of writing instruction occurring across academic disciplines and in a variety of settings. While all of the options may positively impact student writing, (C) is the only one that targets the philosophy of writing across the curriculum.

A leaf falls to earth While butterflies float near. For each, a mirror. Question:To make this poem fit the structure of a classic haiku, the author should change A."A leaf" to "Lone leaf" B."butterflies" to "butterfly" C."float" to "hover" D."mirror" to "birth"

C."float" to "hover" Option (C) is correct. This question asks you to apply your knowledge of literary forms. A classic haiku has five syllables in the first line, seven in the second, and five in the third. The second line in this poem is short one syllable.

During the Second World War, Japanese Americans who lived in the United States were sent to internment camps until the war was over. In her memoir Farewell to Manzanar, Jeanne Wakatsuki tells the story of her internment at one of the camps, Manzanar. The excerpt below describes her first observations upon arriving at the camp with her family at age seven. (1) I could see a few tents set up, the first rows of black barracks, and beyond them, blurred by sand, rows of barracks that seemed to spread for miles across this plain. (2) People were sitting on cartons or milling around, with their backs to the wind, waiting, to see which friends or relatives might be on this bus. (3) As we approached, they turned or stood up, and some moved toward us expectantly. (4) But inside the bus no one stirred. (5) No one waved or spoke. (6) They just stared out the windows, ominously silent. (7) I didn't understand this. (8) Hadn't we finally arrived, our whole family intact? (9) I opened a window, leaned out, and yelled happily, "Hey! This whole bus is full of Wakatsukis!" Question:Which of the following reference materials should students be encouraged to use to help them understand the meaning of "ominously" in sentence 6 ? A.A style manual B.An encyclopedia C.A dictionary D.An almanac

C.A dictionary Option (C) is correct. This question tests your ability to identify the most appropriate reference material for a language-usage task. A dictionary provides word meanings, parts of speech, and etymology, among other things.

I was walking by the Thames. Half-past morning on an autumn day. Sun in a mist. Like an orange in a fried-fish shop. All bright below. Low tide, dusty water and a crooked bar of straw, chicken-boxes, dirt and oil from mud to mud. Like a viper swimming in skim milk. The old serpent, symbol of nature and love. —Joyce Cary, from The Horse's Mouth Which of the following best characterizes the language used to describe the Thames? A.It contains concrete and abstract terms in equal proportion. B.It uses vernacular instead of mythic language to create a sense of place. C.It consistently employs figurative language instead of factual, declarative statements. D.It is conventional and clichéd in its description.

C.It consistently employs figurative language instead of factual, declarative statements. Option (C) is correct. This question tests your knowledge of descriptive conventions. Figurative language is used to describe the sun and the river, as when the sun is compared to "an orange in a fried-fish shop" and the Thames is described as an "old serpent." The passage uses short sentences, but rather than being factual they are figurative in their description of the scene.

If you were to go merely by the quantity of his imitators, you could argue that Dashiell Hammett was a more important writer than James Joyce. He gave his imitators more than an attitude; he gave them a cast of characters, a resilient plot, a setting, a repertory of images, a style, a keyhole view of society, an ethos, and, above all, a hero. Sam Spade is an old American type brought up to date, Hawkeye become private eye with fedora and street smarts instead of leather stockings and wood lore, his turf the last frontier of San Francisco. In the last sentence, the comparison of Sam Spade to Hawkeye alludes to novels by A.Joseph Conrad B.Nathaniel Hawthorne C.James Fenimore Cooper D.Herman Melville

C.James Fenimore Cooper Option (C) is correct. This question tests your ability to recognize names from a well-known author's novels. Both "Hawkeye" and "Leather-Stocking" were nicknames of Natty Bumppo, the pioneer hero of five novels by James Fenimore Cooper (1789-1851), known collectively as the Leather-Stocking Tales.

A high school ELA teacher creates the checklist below based on commonly observed weaknesses in student writing. Writers Check List __ Sentences begin in different ways. __ Sentences build on the ones that precede them. __ Sentences vary in length. __ The meaning of each sentence is clear. __ Sentences flow and are grammatically correct. __ There are no run-on sentences. __ Sentences are complete. The checklist can best be used to monitor which of the following characteristics in students' writing? A.Main ideas B.Writer's voice C.Language conventions D.Organizational structure

C.Language conventions Option (C) is correct. This question tests your ability to recognize the purpose of an assessment tool. The items on the checklist focus on features of the students' writing, such as mechanics, appropriate sentence structure, and correct grammar.

Which of the following sentences is correctly punctuated? A.My favorite works of literature are "The Lottery", The Sun Also Rises, and "Bartleby the Scrivener." B.My favorite works of literature are "The Lottery", The Sun Also Rises, and "Bartleby the Scrivener". C.My favorite works of literature are "The Lottery," The Sun Also Rises, and "Bartleby the Scrivener." D.My favorite works of literature are "The Lottery," The Sun Also Rises, and "Bartleby the Scrivener".

C.My favorite works of literature are "The Lottery," The Sun Also Rises, and "Bartleby the Scrivener." Option (C) is correct. This question requires you to apply your knowledge of the rules of punctuation involving quotation marks. In American usage, commas and periods go inside of quotation marks, whether they are part of the quoted title or not.

(1) How does the work of the journalist differ from that of the historian? (2) The most obvious difference is that the journalist reports the everyday triumphs and follies of the human race and has little time to reflect on these occurrences, whereas the historian must distinguish between what is trivial and what is important in both the past and the present. (3) As a journalist, I report as news today stories that may or may not be treated as history in later generations. (4) The historian will sort through masses of information and create what it is hoped will be a graceful accounting of what happened, why, and with what result. Revising which of the following sentences would be necessary in order to put the passage entirely in the third person? A.Sentence 1 B.Sentence 2 C.Sentence 3 D.Sentence 4

C.Sentence 3 Option (C) is correct. In third-person point of view, the author does not refer to himself or herself, suggesting objectivity in regard to the topic. In Sentence 3, the writer refers to himself using the personal pronoun "I" ("As a journalist, I report as news today..."), which means that the passage is written in the first person.

After students have completed reading a class novel, the teacher gives students three broad questions that require them to evaluate and make decisions about the novel's theme and characters. Students are given time to prepare answers to the questions, and then they discuss the questions as a class. The teacher acts as a facilitator during the discussion, but the actual discussion is primarily student led. The scenario best describes which of the following discussion models? A.Literature circles B.Think-pair-share C.Socratic seminar D.Reciprocal teaching

C.Socratic seminar Option (C) is correct. This question tests your ability to understand different models of effective oral communication. Socratic seminars are student-led discussions that give all students the chance to share their thoughts about the novel. They also allow students to question the text, which is part of Ms. White's goal. Socratic seminar is a formal discussion, based on a text, in which the leader asks open-ended questions. Within the context of the discussion, students listen closely to the comments of others, think critically for themselves, and articulate their thoughts and responses to the thoughts and responses of others.

Which of the following is an important principle of process writing? A.It is appropriate for students to eliminate stages in the process as they become practiced writers. B.Some modes of writing are more important than others. C.Students may need to move back and forth through the writing stages. D.The teacher is the best audience for student writing.

C.Students may need to move back and forth through the writing stages. Option (C) is correct. This question asks you to recognize a basic principle of process writing. The stages of the writing process are recursive. After editing, a student may go back to revise a piece even more. Furthermore, the revision may necessitate a prewriting strategy to generate more content.

Which of the following lists contains words that are pronounced differently depending on whether they are used as nouns or verbs? A.Paper, supply, chronicle B.Table, herd, carpet C.Suspect, conscript, present D.Drive, catalog, board

C.Suspect, conscript, present Option (C) is correct. This question tests your knowledge of parts of speech and how certain words can function as more than one part of speech; in this case, with a different pronunciation accompanying the functional shift.

Question:Which of the following best defines the term "vernacular language"? A.A simplified language used between two or more groups B.A stable natural language developed from a parent language C.The dialect, or normal spoken form of language, of a specific population D.The use of informal words and expressions that are not considered standard

C.The dialect, or normal spoken form of language, of a specific population Option (C) is correct. This question tests your knowledge of linguistic terms. A vernacular is the native language or native dialect of a specific population.

Two political activists from opposite sides of an issue are invited to speak to a teacher's eleventh-grade English class. Which of the following describes the activity that would be most effective for the teacher to use to encourage students to evaluate each speaker's point of view, reasoning, and use of evidence and rhetoric? A.The students research the issue on their own after the speakers visit, and write papers in which they take a stand on one side of the issue. B.Before the presentation, the students work in groups to prepare questions about the issue to ask the speakers. C.The teacher records the presentation and asks students to review the presentation in small groups, then present an analysis of the speakers' arguments to the class. D.After the presentation, the students have a debate in which they must present evidence and use logic to persuade the audience of their arguments.

C.The teacher records the presentation and asks students to review the presentation in small groups, then present an analysis of the speakers' arguments to the class. Option (C) is correct. This question tests your ability to recognize appropriate classroom activities to accompany oral presentations. Because oral presentations go by quickly, it can be useful to review presentations and collaborate with peers to analyze their effectiveness.

Question:Which of the following strategies should a teacher recommend that students apply to best help them select credible sources for a research project? A.Limiting sources strictly to print publications B.Excluding sources with content that is controversial or unpleasant C.Verifying information they obtain against that in other sources D.Grouping sources into specific subcategories

C.Verifying information they obtain against that in other sources Option (C) is correct. Information should always be checked against other sources to ensure credibility.

The following excerpt from Richard Rodriguez' Hunger of Memory: The Education of Richard Rodriguez describes the author's experience of taking reading lessons. Everything about our sessions pleased me: the smallness of the room; the noise of the janitor's broom hitting the edge of the long hallway outside the door; the green of the sun, lighting the wall. . . . Most of the time we took turns. I began with my elementary text. Sentences of astonishing simplicity seemed to me lifeless and drab: "The boys ran from the rain . . . She wanted to sing . . . The kite rose in the blue." Then the old nun would read from her favorite books, usually biographies of early American presidents. Playfully she ran through complex sentences, calling the words alive with her voice, making it seem that the author somehow was speaking directly to me. I smiled just to listen to her. I sat there and sensed for the very first time some possibility of fellowship between a reader and a writer, a communication, never intimate like that I heard spoken words at home convey, but one nonetheless personal. Question:It can be inferred that Rodriguez found the nun's way of reading to be A.amusing but somewhat tedious B.exhilarating but hard to understand C.appealing and illuminating D.monotonous and wearisome

C.appealing and illuminating Option (C) is correct. The author is in awe of the nun's way of reading. He says, "Playfully she ran through complex sentences, calling the words alive with her voice, making it seem that the author somehow was speaking directly to me." Her reading helped him to realize for the first time the personal connection between a reader and a writer. Therefore, it is evident that he found the nun's way of reading appealing and illuminating.

(1) The overwhelming majority of immigrants to America in the nineteenth century settled in the northeastern and north-central states. (2) The impact of their numbers was staggering. (3) In 1890 New York City contained twice as many Irish as Dublin, the same number of Germans as Hamburg, half as many Italians as Naples, and two and one-half times the Jewish population of Warsaw. (4) Many of these recent immigrants had come willingly, while others had been forced out of their home countries by overpopulation, crop failure, famine, violence, or industrial depression. The information in sentence 3 would best be expressed graphically in the form of a A.map B.pie chart C.bar graph D.T-chart

C.bar graph Option (C) is the correct. This question asks you to identify the best way of graphically representing specific information. Sentence 3 compares the ethnic populations of New York City to those of major European cities from which the immigrants came. Because it can visually represent the relative size of each group living in New York and in the referenced European city, the information could best be represented as a bar graph.

So your customers have ordered a lot of your widgets online. Congratulations! But don't start counting your chickens: You still have some work to do before you make those sales. You can't send customers their products until you calculate taxes and shipping. And more importantly, you still need to determine how to process the customers' preferred methods of payment. The key is to figure out the best software solution for your situation. Question:The word in the passage that most clearly indicates the desired audience is A.calculate B.preferred C.customers D.widgets

C.customers Option (C) is correct. This question asks you to identify a word from the passage that best enables you to identify the intended audience. As can be seen from phrases such as "your customers," "make those sales," and "for your situation," the intended audience is people who are engaged in selling goods to customers.

The passage below is taken from N. Scott Momaday's The Way to Rainy Mountain, a collection of Kiowa legends. A single knoll rises out of the plain in Oklahoma, north and west of the Wichita Range. For my people, the Kiowas, it is an old landmark, and they gave it the name Rainy Mountain. The hardest weather in the world is there. Winter brings blizzards, hot tornadic winds arise in the spring, and in summer the prairie is an anvil's edge. The grass turns brittle and brown, and it cracks beneath your feet. There are green belts along the rivers and creeks, linear groves of hickory and pecan, willow and witch hazel. At a distance in July or August the steaming foliage seems almost to writhe in fire. Great green and yellow grasshoppers are everywhere in the tall grass, popping up like corn to sting the flesh, and tortoises crawl about on the red earth, going nowhere in the plenty of time. Loneliness is an aspect of the land. All things in the plain are isolate; there is no confusion of objects in the eye, but one hill or one tree or one man. To look upon that landscape in the early morning, with the sun at your back, is to lose the sense of proportion. Question:The principal rhetorical strategy Momaday uses in the passage is A.narration B.definition C.description D.analysis of process

C.description Option (C) is correct. This question tests your ability to identify rhetorical strategies. Rhetorical strategies include the patterns authors use to develop paragraphs. This paragraph is developed through description — the systematic use of details of the landscape at various times of year and in various aspects.

My sister, Mrs. Joe Gargery, was more than twenty years older than I, and had established a great reputation with herself and her neighbours because she had brought me up "by hand." Having at that time to find out for myself what the expression meant, and knowing her to have a hard and heavy hand, and to be much in the habit of laying it upon her husband as well as upon me, I supposed that Joe Gargery and I were both brought up by hand. She was not a good-looking woman, my sister, and I had a general impression that she must have made Joe Gargery marry her by hand. Joe was a fair man, with curls of flaxen hair on each side of his smooth face, and with eyes of such a very undecided blue that they seemed to have somehow got mixed with their own whites. He was a mild, good-natured, sweet-tempered, easy-going, foolish, dear fellow—a sort of Hercules in strength and also in weakness. —Charles Dickens, from Great Expectations In the passage, the character of Mrs. Gargery is developed primarily through the author's depiction of her A.husband's opinion of her B.most prominent physical traits C.effect on other people D.reputation among her neighbors

C.effect on other people Option (C) is correct. This question tests your knowledge of techniques of characterization. In this passage, the reader learns about Mrs. Gargery mainly in two ways: first, through the naive narrator's conjecture that the phrase "by hand" refers to her habit of clouting the boy and her husband; and, second, through the indications in the second paragraph that her husband has always submitted to her will. Mrs. Gargery is described briefly: "She was not a good-looking woman." But in this passage, her sharp temper and the intimidating force of her character are communicated by her effect on the narrator and on her husband, Joe.

It took well over a year. I don't mean for us. I mean for the animals. Considering that animals dispense with clothes, footwear, linen, furniture, kitchenware, toiletries; that nationality means nothing to them; that they care not a jot for passports, money, employment prospects, schools, cost of housing, healthcare facilities—considering, in short, their lightness of being, it's amazing how hard it is to move them. Moving a zoo is like moving a city. —Yann Martel, The Life of Pi The author uses a semicolon between "toiletries" and "that" ("toiletries; that nationality") most likely in order to A.link two independent clauses B.link two prepositional phrases C.link lists in which items contain commas D.take the place of a conjunctive adverb

C.link lists in which items contain commas Option (C) is correct. This question tests your knowledge of the use of the semicolon in a sentence. One correct use of the semicolon is to use it to separate lists of items that are separated by commas. The semicolon is used to avoid confusion.

Our watch will adjust the date automatically for the next 50 years, even during leap years. It will stay accurate to within 10 seconds a year, no matter how many times you adjust it for time zone changes. And it requires you to replace the battery just once a decade. There's only one conclusion — you should change your thinking about watches. Question:The advertising strategy most present in the passage is A.describing the product's elegance and beauty B.making an affective appeal to potential buyers C.presenting the product's range of special features D.comparing the product with that of a competitor

C.presenting the product's range of special features Option (C) is correct. This question asks you to identify the advertising strategy most present in the passage. Three of the four sentences in the passage itemize the watch's special features.

Federalism refer to the division of government powers between the national and state governments. The author of the sentence above will most likely benefit from a review of A.prepositions B.capitalization C.verb number D.parallelism

C.verb number Option (C) is correct. This item tests your knowledge of subject-verb agreement and appropriate sentence structure. The sentence contains a subject-verb agreement error between "Federalism" (a singular noun) and "refer" (a plural verb).

Consider the four characteristics listed below. Identify which of the characteristics apply to each of the two types of text. For each row, select all that apply. Some rows may be empty. Characteristic Functional Texts Expository Texts Their purpose is to inform. 1 2 Their purpose is to persuade. 3 4 They include lists of instructions. 5 6 They often discus causes and effects. 7 8

Correct Answer: 1, 2, 5, 8 Correct options are explained in the following paragraph. The purpose of both functional and expository texts is to inform, not to persuade, but functional texts are more focused on supporting the completion of everyday tasks, so they include lists of instructions. Expository essays can follow many organizational patterns, including the cause-effect structure for discussing one kind of relationship between concepts.

A student is searching for sources for a research project about former president Ronald Reagan. For each of the sources listed below, check the box to indicate whether the source is a primary source or a secondary source. Primary Source Secondary Source The suit Ronald Reagan wore to his inauguration 1 2 A transcript of a speech given by Ronald Reagan 3 4 A business journal article about Ronald Reagan 5 6 A commissioned photograph of Ronald Reagan 7 8

Correct Answer: 1, 3, 6, 7 Options (1), (3), (6), and (7) are correct. A primary source is a document or physical object that was written or created during the time under study. Such sources were present during an experience or time period and offer an inside view of a particular event. The suit that Reagan wore (1), the transcript of a speech that he gave (3), and the photograph (7) fit the criteria in that they are original artifacts and documents from his presidency. A secondary source interprets and analyzes primary sources. A secondary source is one or more steps removed from the event. Secondary sources may contain pictures, quotes, or graphics of primary sources. A business journal article about Reagan (6) has synthesized information from many sources and so is a secondary source.

To which of the following sources should a teacher direct a student who is looking for a credible source for a research paper? A. An editorial column in an online newspaper B. A series of blog posts by a university professor C. A wiki encyclopedia article D. An online peer-reviewed journal

D. An online peer-reviewed journal Option (D) is correct. This question tests your ability to evaluate sources. A peer-reviewed journal is evaluated by other experts in the field and is considered a credible source of information. While a university professor may be an expert in his or her field, an informal, unreviewed blog post may contain biased or incorrect information.

A 10-minute free-writing activity is best used for which of the following purposes? A. Organizing instructions from a procedural text into steps B. Gathering textual evidence for a review of a literary text C. Compiling data to support arguments in a persuasive essay D. Generating ideas for elaborating on a personal narrative

D. Generating ideas for elaborating on a personal narrative Option (D) is correct. This question tests your ability to recognize research-based strategies for teaching writing. Free writing is a strategy designed to help students generate ideas by spending a predetermined amount of time writing everything that comes to mind about a topic. Afterward students can examine the results of their free writing and gain ideas about what can be added as elaboration to their personal narratives.

In the Gobi Desert, scientists recently discovered some of the world's most extensive fossil fields, which yielded the remnants not only of dinosaurs and their eggs and also plants previously unknown to science. Question:Which of the following versions of the underlined portion will yield a correct sentence? A. in addition to B. as well as finding that of C. they also found D. but also of

D. but also of Option (D) is correct. The correct correlative conjunction for "not only" is "but also," and this is the only option that works in the context of the sentence.

Which of the following sentences most likely comes from a piece of writing that defends a particular political view? A."Humans have been exploring space for only a short time, yet the impact on humanity has been vast." B."A three-hour debate, televised nationwide, would add considerable weight to the issue." C."The commercial logic behind such deals centers around the need for companies to refine business practices." D."A strong and capable military is the foundation of the peace we enjoy today."

D."A strong and capable military is the foundation of the peace we enjoy today." Option (D) is correct. This question asks you to identify the sentence that is most likely to come from a piece of writing that is designed to express a political view. Of the four topics addressed in the sentences — space exploration, a political debate, business deals, and the military — the final sentence most directly addresses a political view.

Any use of language that calls a fire in a nuclear reactor building "rapid oxidation," the illegal overthrow of a legitimate government "destabilizing a government," crimes "inappropriate actions," and lies "inoperative statements" is language that attempts to avoid responsibility, that attempts to make the bad seem good, the negative appear positive, something unpleasant appear attractive, and that seems to communicate but does not. It is language designed to alter our perception of reality and corrupt our minds. Such language does not provide the tools needed to develop and preserve civilization. Such language breeds suspicion, cynicism, distrust, and ultimately, hostility. Question:Which of the following is an example of language use similar to that in the quoted phrases within the passage? A."Performer" for musician or dancer B."Let the cat out of the bag" for giving away a secret C."Opening Pandora's box" for asking a question that raises many problematical issues D."Rectification of boundaries" for invasion

D."Rectification of boundaries" for invasion Option (D) is correct. This question tests your knowledge of semantics, especially euphemism. The quoted phrases in the passage illustrate the use of language to hide rather than reveal the realities to which it refers. Similarly, the phrase "rectification of boundaries" in (D) fails to capture the hostility and unlawfulness of an invasion.

Mr. Scott's students have repeatedly inserted information from websites into their research papers without considering the websites' credibility. Which of the following URL extensions should Mr. Scott tell his students best indicates a credible website? A..net B..com C..org D..gov

D..gov Option (D) is correct. This question tests your ability to evaluate the credibility of digital sources. Of the choices, .gov websites are the most reliable because they cannot be purchased as domains by the public, which means that they are regulated by government rules and are, therefore, subject to strict guidelines.

During the Second World War, Japanese Americans who lived in the United States were sent to internment camps until the war was over. In her memoir Farewell to Manzanar, Jeanne Wakatsuki tells the story of her internment at one of the camps, Manzanar. The excerpt below describes her first observations upon arriving at the camp with her family at age seven. (1) I could see a few tents set up, the first rows of black barracks, and beyond them, blurred by sand, rows of barracks that seemed to spread for miles across this plain. (2) People were sitting on cartons or milling around, with their backs to the wind, waiting, to see which friends or relatives might be on this bus. (3) As we approached, they turned or stood up, and some moved toward us expectantly. (4) But inside the bus no one stirred. (5) No one waved or spoke. (6) They just stared out the windows, ominously silent. (7) I didn't understand this. (8) Hadn't we finally arrived, our whole family intact? (9) I opened a window, leaned out, and yelled happily, "Hey! This whole bus is full of Wakatsukis!" A teacher is having students collaborate on inquiry projects based on Farewell to Manzanar. Which of the following tools is most appropriate for the students to use to compile and edit their research online? A.A blog B.A mind map C.A digital story D.A wiki

D.A wiki Option (D) is correct. This question tests your ability to recognize the most appropriate technological tool for a task. A wiki is an online tool for collaboratively creating and editing content.

Just then something long, round, limp and black fell upon her shoulders and slithered to the floor beside her. A great terror took hold of her. It softened her knees and dried her mouth so that it was a full minute before she could cry out or move. Then she saw that it was the big bull whip her husband liked to carry when he drove. She lifted her eyes to the door and saw him standing there bent over with laughter at her fright. She screamed at him. "Sykes, what you throw dat whip on me like dat? You know it would skeer me—looks just like a snake, an' you knows how skeered Ah is of snakes." —Zora Neale Hurston, from the short story "Sweat"(1926) A teacher is writing a lesson on dialect using a passage from the short story. "Sykes, what you throw dat whip on me like dat? You know it would skeer me—looks just like a snake, an' you knows how skeered Ah is of snakes." Question:Which of the following strategies would be most effective in helping students understand the meaning of the words italicized in the sentence above? A.Having the students use a dictionary to look up the words B.Giving the students a teacher-made glossary of African American dialect used in the story C.Preteaching the word variations in the dialect before having the students read the text D.Asking the students to search for clues elsewhere in the passage that might explain the meaning of the words

D.Asking the students to search for clues elsewhere in the passage that might explain the meaning of the words Option (D) is correct. This question tests your ability to recognize reading comprehension strategies. In the text, "dat" means "that." By reading the sentence carefully, students will see that "dat" is functioning as a demonstrative adjective. "Skeered" means "scared." The word has a similar sound as "scared," and the reader already knows that the character is scared of snakes.

I was a child and she was a child, In this kingdom by the sea; But we loved with a love that was more than love— I and my Annabel Lee; With a love that the wingéd seraphs of heaven Coveted her and me. Question:The excerpt is from a poem written by which of the following authors? A.William Shakespeare B.John Keats C.Henry Wadsworth Longfellow D.Edgar Allan Poe

D.Edgar Allan Poe Option (D) is correct. Edgar Allan Poe wrote "Annabel Lee" about a lost love.

All the cucurbit crops can be attacked; the squash bug shows a preference for squashes and pumpkins. Squash bugs damage plants by removing sap and causing leaves to wilt and collapse. They are the vector of newly recognized disease of cucurbit crops, yellow vine decline. Early infection by the bacteria that causes the disease can result in severe yield loss, so it is very important to prevent squash bugs from feeding on young plants early in the season. Adult squash bugs begin to fly into fields and gardens in late May and early June. What would be the most effective title for Felix's slide? A.Diseases Caused by Squash Beetles B.Examples of Effects of Summer Insects C.Summer Cucurbit Susceptibility to Squash Beetles D.Effects of Squash Beetles on Cucurbits

D.Effects of Squash Beetles on Cucurbits Option (D) is correct. This question tests your ability to recognize qualities of an effective presentation. An effective title avoids unnecessary information and accurately represents the information on the slide.

A. For thou hast sent her a mantle of green, As green as any grass, And bade her come to the silver wood To hunt with Child Maurice. B. An Ace of Hearts steps forth: the King unseen Lurked in her hand, and mourned his captive Queen: He springs to vengeance with an eager pace, And falls like thunder on the prostrate Ace. The nymph exulting fills with shouts the sky; The walls, the woods, and long canals reply. C. Who would have thought my shriveled heart Could have recovered greenness? It was gone Quite underground; as flowers depart To see their mother-root, when they have blown, Were they together All the hard weather, Dead to the world, keep house unknown. D. But wherefore rough, why cold and ill at ease? Aha, that is a question! Ask, for that, What knows,—the something over Setebos That made Him, or He, may be, found and fought, Worsted, drove off and did to nothing, perchance. Which is from a dramatic monologue? A.Excerpt A B.Excerpt B C.Excerpt C D.Excerpt D

D.Excerpt D Option (D) is correct. This question tests your knowledge of literary genres. The lines of a dramatic monologue are spoken by a character whose personality, motives, and circumstances shape the way he or she tells a story and can, in turn, be inferred from the story told. The lines of (D) are from the dramatic monologue "Caliban upon Setebos" by Robert Browning.

Just then something long, round, limp and black fell upon her shoulders and slithered to the floor beside her. A great terror took hold of her. It softened her knees and dried her mouth so that it was a full minute before she could cry out or move. Then she saw that it was the big bull whip her husband liked to carry when he drove. She lifted her eyes to the door and saw him standing there bent over with laughter at her fright. She screamed at him. "Sykes, what you throw dat whip on me like dat? You know it would skeer me—looks just like a snake, an' you knows how skeered Ah is of snakes." —Zora Neale Hurston, from the short story "Sweat"(1926) Which of the following questions would most likely be asked by a psychoanalytical critic when analyzing the passage? A.How do the events present in the work reflect the current events of the author's day? B.How does this text use imagery to develop its theme? C.How do the sounds of the words as they are spoken by the reader enhance or change the meaning of the text? D.How does the family dynamic affect the character's actions?

D.How does the family dynamic affect the character's actions? Option (D) is correct. This question tests your ability to recognize a variety of literary criticisms. The analysis is approached by dissecting the underlying psychology of the characters, which is characteristic of a psychoanalytical critic.

In an editorial about recycling, an author writes, "My parents, who only have two people in their household, consume enough bottled water in one week to quench the thirst of the entire state of Rhode Island. Not surprisingly, they do not recycle the bottles!" The author uses which of the following rhetorical strategies? A.Satire B.Irony C.Understatement D.Hyperbole

D.Hyperbole Option (D) is correct. This question tests your ability to recognize rhetorical strategies that authors use in texts. The sentence, "My parents . . . consume enough water in one week to quench the thirst of the entire state of Rhode Island!" is an exaggeration, which is the definition of hyperbole.

Any use of language that calls a fire in a nuclear reactor building "rapid oxidation," the illegal overthrow of a legitimate government "destabilizing a government," crimes "inappropriate actions," and lies "inoperative statements" is language that attempts to avoid responsibility, that attempts to make the bad seem good, the negative appear positive, something unpleasant appear attractive, and that seems to communicate but does not. It is language designed to alter our perception of reality and corrupt our minds. Such language does not provide the tools needed to develop and preserve civilization. Such language breeds suspicion, cynicism, distrust, and ultimately, hostility. Which of the following statements would best follow the last sentence to complete the passage? A.The effects of such language on a culture, if deleterious, are short-term and minimal. B.However, it does not present a serious problem because it is used and read by only a narrow segment of society. C.It reflects an intellectually appropriate application of ingenuity in treating language as a living and changing organism. D.It is insidious, because it can infect and, ultimately, destroy the function of language — communication among people and social groups.

D.It is insidious, because it can infect and, ultimately, destroy the function of language — communication among people and social groups. Option (D) is correct. This question tests your ability to complete an argument about semantic practices. The statement in (D) brings the paragraph to synthesis and conclusion by delineating the ultimate effect of the degradation of the language described in the previous few sentences. The degradation of language is condemned throughout the paragraph; none of the other choices preserves the unity of logic of the paragraph.

Which of the following follows the proper MLA format for parenthetical citations? A.Jones believes that Vivaldi deserves credit for more than just his prolific output (Jones, 157). B.Jones (157) believes that Vivaldi deserves credit for more than just his prolific output. C.Jones believes that Vivaldi deserves credit for more than just his prolific output. (157) D.Jones believes that Vivaldi deserves credit for more than just his prolific output (157).

D.Jones believes that Vivaldi deserves credit for more than just his prolific output (157). Option (D) is correct. This question asks you to apply your knowledge of documentation techniques. Since the author's name is in the text, it is not used in the reference. The page number follows the information, but comes before the period.

A class is creating a blog about ways to get involved in the community. Students will take turns updating the blog throughout the school year. Question:Which of the following lessons will target the objective of using the blog as a tool for students to interact with each other through technology? A.Editing and inserting visual images B.Inserting a link to a different site C.Incorporating the previous post D.Leaving and responding to comments

D.Leaving and responding to comments Option (D) is correct. This question tests your ability to identify strategies for encouraging meaningful use of technology in the classroom. A blog is a website that contains posts and comments, where comments represent a dynamic mode of communication between author and audience. Although incorporating the previous post into a comment represents a form of interaction between the writer and the reader, it is limited and one-sided.

All the cucurbit crops can be attacked; the squash bug shows a preference for squashes and pumpkins. Squash bugs damage plants by removing sap and causing leaves to wilt and collapse. They are the vector of newly recognized disease of cucurbit crops, yellow vine decline. Early infection by the bacteria that causes the disease can result in severe yield loss, so it is very important to prevent squash bugs from feeding on young plants early in the season. Adult squash bugs begin to fly into fields and gardens in late May and early June. Question:Felix is creating a slide show presentation and wants to improve the quality of this slide. What is the most effective way for him to improve the readability of the slide? A.Aligning the text flush left B.Using colors that contrast C.Using a font that is easy to read D.Limiting information to essentials

D.Limiting information to essentials Option (D) is correct. This question tests your ability to recognize qualities of an effective presentation. Limiting the information on the slide to essentials will make it much easier for the audience to read because it will decrease the visual load and will make important information stand out. Although left-aligning the text, adding contrasting colors, and choosing a font that is easy to read may add to the readability of the slide, the best choice is (D).

(1) The overwhelming majority of immigrants to America in the nineteenth century settled in the northeastern and north-central states. (2) The impact of their numbers was staggering. (3) In 1890 New York City contained twice as many Irish as Dublin, the same number of Germans as Hamburg, half as many Italians as Naples, and two and one-half times the Jewish population of Warsaw. (4) Many of these recent immigrants had come willingly, while others had been forced out of their home countries by overpopulation, crop failure, famine, violence, or industrial depression. In which of the following sources would a student be most likely to find further evidence to support the claims made in sentence 4 ? A.Film footage of immigrants arriving in the United States B.Website search engines C.Early-twentieth-century American novels D.Primary sources, such as journals and letters written by immigrants

D.Primary sources, such as journals and letters written by immigrants Option (D) is correct. This question tests your knowledge of appropriate research sources for particular goals. Sentence 4 makes a claim about the immigrants' motives for migration. Primary sources would be most likely to reveal the immigrants' motives.

A high school teacher wants to meet the needs of reluctant readers who are required to read Dickens' A Tale of Two Cities. Which of the following materials is most likely to motivate these students to become engaged in the book? A.A book about Dickens' country and city of origin B.Photographs of the town in which Dickens wrote the novel C.Copies of the novel that are printed in large, easy-to-read print D.Software allowing students to interact with the setting of the novel

D.Software allowing students to interact with the setting of the novel Option (D) is correct. This question asks you to identify a frequently effective way to foster appreciation of reading among students in grades 7-12. (A) and (B), historical in nature and noninteractive, would probably not stimulate many students to make the transition from reluctant to interested. A change in format, reflected in (C), would be unlikely to change students' attitudes. Students often become interested in reading a book if they can interact with aspects of the setting of the book — in the case of Dickens and eighteenth century France and England.

A hush is over all the teeming lists, And there is pause, a breath-space in the strife; A spirit brave has passed beyond the mists And vapors that obscure the sun of life. And Ethiopia, with bosom torn, Laments the passing of her noblest born. For her his voice, a fearless clarion, rung That broke in warning on the ears of men; For her the strong bow of his power he strung, And sent his arrows to the very den Where grim Oppression held his bloody place And gloated o'er the mis'ries of a race. Through good and ill report he cleaved his way Right onward, with his face set toward the heights, Nor feared to face the foeman's dread array,— The lash of scorn, the sting of petty spites. He dared the lightning in the lightning's track, And answered thunder with his thunder back. —Paul Laurence Dunbar, from "Frederick Douglass" A. Student A: This poem is about war. It's about "strife" or "bloody" fighting. The champion "cleaved his way Right onward." B. Student B: A famous leader is praised for all the contributions he made to his people. He met much oppression in his struggle for his people but overcame it all. C. Student C: This lady, Ethiopia, is very sad when someone close to her dies. This person, who was killed in war, helped people. He must have been some kind of a super warrior or maybe even a god. D. Student D: It interested me the way this poem is so concerned with sound and noise. It begins on a quiet note and death but then builds up to the final stanza which is full of sound words and action. Question:Which student response points out the author's appeal to the senses to complement the content of the poem? A.Student A B.Student B C.Student C D.Student D

D.Student D Option (D) is correct. This question asks you identify the use of a particular descriptive approach. Student D, noting how the poem moves from a "quiet note" to an ending "full of sound words," focuses on the author's appeal to the senses, particularly the sense of hearing. Students A, B, and C are more concerned with the poem's theme or story than with how the poem achieves its effects.

A television advertisement promoting the benefits of an over-the-counter pain reliever begins by depicting scenes from the lives of satisfied consumers and concludes with a licensed physician endorsing the drug's positive effects. The persuasive strategy most evident in the advertisement is A.Emotional appeal B.Loaded words C.Snob appeal D.Testimonial

D.Testimonial Option (D) is correct. This question tests your ability to recognize persuasive techniques in advertisements. Testimonial is a strategy where a celebrity, an expert, or a satisfied customer endorses a product or an idea. In the advertisement described, a licensed physician is endorsing the positive effects of the medicine.

Which of the following criteria would be LEAST important in evaluating the information provided in an expository article from an Internet source? A.The date the source was written or last updated B.The comprehensiveness of the information provided C.The credibility of the author D.The number of links to other Internet sites

D.The number of links to other Internet sites Option (D) is correct. The question refers to evaluating the information in the article, not the website it is from. The number of links available would help in determining the helpfulness of the Internet site but would be unrelated to the relevance, comprehensiveness, or credibility of the article.

Just then something long, round, limp and black fell upon her shoulders and slithered to the floor beside her. A great terror took hold of her. It softened her knees and dried her mouth so that it was a full minute before she could cry out or move. Then she saw that it was the big bull whip her husband liked to carry when he drove. She lifted her eyes to the door and saw him standing there bent over with laughter at her fright. She screamed at him. "Sykes, what you throw dat whip on me like dat? You know it would skeer me—looks just like a snake, an' you knows how skeered Ah is of snakes." —Zora Neale Hurston, from the short story "Sweat"(1926) Based on the excerpt, which of the following statements is true? A.The story is told from first-person point of view. B.The story is told from second-person point of view. C.The story is told from a third-person objective point of view. D.The story is told from a third-person omniscient point of view.

D.The story is told from a third-person omniscient point of view. Option (D) is correct. This question tests your ability to recognize point of view in a narrative. The narrator conveys both what is happening in the scene and how the character is feeling, which is characteristic of the omniscient point of view.

(1) People have debated the best ways to educate children for decades. (2) One of the many aspects of education that is often debated is the structure of the school years. (3) Historically, students were given summers off so they could help their families harvest crops. (4) As more families got away from farming, this became less necessary, but many argued that money is saved by closing schools for the summer. (5) With other countries surpassing the test scores of American students, the concept of year-round schooling is again being pushed by lawmakers. Which of the following reference materials would be most appropriate in helping the student avoid the redundancy of using the word "debated" in both sentence 1 and sentence 2 ? A.Online glossary B.Spell-checker C.Dictionary D.Thesaurus

D.Thesaurus Option (D) is correct. This question tests your knowledge of reference materials. A thesaurus would help the student find a synonym for debate.

A key difference between written and oral discourse is that in written discourse the writer A.frequently writes without a specific audience in mind B.is less likely to use conjunctions to link ideas together C.is more likely to present a cohesive and well-organized argument D.cannot rely on elements of paralanguage to reveal attitude

D.cannot rely on elements of paralanguage to reveal attitude Option (D) is correct. Paralanguage is often used to refer to elements of speech, such as prosody, pitch, volume, and intonation, that suggest meaning or convey emotion. These cannot be relayed in written discourse.

If you were to go merely by the quantity of his imitators, you could argue that Dashiell Hammett was a more important writer than James Joyce. He gave his imitators more than an attitude; he gave them a cast of characters, a resilient plot, a setting, a repertory of images, a style, a keyhole view of society, an ethos, and, above all, a hero. Sam Spade is an old American type brought up to date, Hawkeye become private eye with fedora and street smarts instead of leather stockings and wood lore, his turf the last frontier of San Francisco. Question:The passage discusses the A.shortcomings of authors who lack formal training B.value of continuing to teach the classics C.characteristics of writers who are essentially derivative and unimaginative D.creation of a new popular genre Correct

D.creation of a new popular genre Option (D) is correct. This question tests your ability to comprehend the main idea of a passage. The passage asserts that Hammett's imitators wrote novels similar to his and to one another's, thereby creating a body of work that constitutes a genre. That Hammett was widely copied — the passage mentions a "quantity of his imitators" — indicates that the genre was also a popular one.

"He was a most disagreeable companion. . . . His conversation was a procession of one." Question:This comment by Florence Nightingale about an unidentified man shows most clearly that she is A.angered by his condescending attitude B.intimidated by his temper C.dismayed by his appearance D.critical of his vanity

D.critical of his vanity Option (D) is correct. This question asks you to identify Florence Nightingale's criticism of an unnamed man. Nightingale states that his conversation was a "procession of one"; that is, that he alone was the subject of the conversation. People who talk only about themselves exhibit vanity.

Him the Almighty Power Hurled headlong flaming from the ethereal sky, With hideous ruin and combustion, down To bottomless perdition; there to dwell In adamantine chains and penal fire, Who durst defy the Omnipotent to arms. Question:Milton places "Him" first in the sentence to A.be consistent with the typical seventeenth-century English object-verb pattern B.serve as an appositive to "the Almighty Power" C.focus attention on the character performing the action D.focus attention on the character receiving the action

D.focus attention on the character receiving the action Option (D) is correct. This question tests your ability to interpret a poetic strategy. Sentences in both seventeenth-century and modern English most commonly follow a subject-verb-object pattern. Variations of this word order can, by contrast, stand out and command the reader's attention. In Milton's sentence, the character referred to by the objective pronoun "Him" is the object receiving the action performed by the "Almighty Power." Placing the objective pronoun in the initial position in the sentence has the effect of calling attention to the character represented by the pronoun — not only because the usual pattern of English has been altered, but also because the reader encounters that word before any others.

Students in a high school English Language Arts class are divided into small groups to discuss an expository text that they recently read. To best encourage students to initiate discussion within their groups, the teacher should provide students with a A.summary of the main ideas of the text B.copy of the text for each group member C.rubric to assess the group's performance D.list of sample questions relating to the text

D.list of sample questions relating to the text Option (D) is correct. This question tests your ability to recognize strategies for promoting effective classroom discussions. Students often have difficulty knowing how to begin discussing a text in a small group. Providing a list of sample questions gives the discussion an initial focus and purpose on which students can build the discussion.

The passage below is taken from N. Scott Momaday's The Way to Rainy Mountain, a collection of Kiowa legends. A single knoll rises out of the plain in Oklahoma, north and west of the Wichita Range. For my people, the Kiowas, it is an old landmark, and they gave it the name Rainy Mountain. The hardest weather in the world is there. Winter brings blizzards, hot tornadic winds arise in the spring, and in summer the prairie is an anvil's edge. The grass turns brittle and brown, and it cracks beneath your feet. There are green belts along the rivers and creeks, linear groves of hickory and pecan, willow and witch hazel. At a distance in July or August the steaming foliage seems almost to writhe in fire. Great green and yellow grasshoppers are everywhere in the tall grass, popping up like corn to sting the flesh, and tortoises crawl about on the red earth, going nowhere in the plenty of time. Loneliness is an aspect of the land. All things in the plain are isolate; there is no confusion of objects in the eye, but one hill or one tree or one man. To look upon that landscape in the early morning, with the sun at your back, is to lose the sense of proportion. "Great green and yellow grasshoppers are everywhere in the tall grass, popping up like corn to sting the flesh, and tortoises crawl about on the red earth, going nowhere in the plenty of time." This sentence contains an example of A.comma splices B.passive voice C.two relative clauses D.parallel structure

D.parallel structure Option (D) is correct. This question tests your ability to identify grammatical elements. "Popping up like corn" and "going nowhere in the plenty of time" are examples of participial phrases used in parallel ways within the sentence.

The previously unpublished writers in this anthology are part of a recent phenomenon—in their numbers probably a unique phenomenon in history. They are not an organized movement; they don't have a common politics; they don't even have a name. Are they "new/old" writers or "promising old" writers—that juxtaposition of adjectives is peculiar to the ears; are they "late bloomers" or simply late-developing writers? —Sondra Zeidenstein, from A Wider Giving: Women Writing after a Long Silence The first sentence of the passage suggests that the writers in the anthology are part of "a unique phenomenon in history" because A.their work has not been published before B.their work has been unappreciated for a long time C.they are not affiliated with any organized movement D.they are members of a relatively large group

D.they are members of a relatively large group Option (D) is correct. The phrase "in their numbers probably a unique phenomenon in history" indicates that the "recent phenomenon" of previously unpublished writers having work published later in life is unique because in the past the number of writers published later in life was smaller. The fact that the writers' work has not been published before does not make them unique. Although the writers' work may have been unappreciated for a long time because it was unpublished, according to the passage, this is not the phenomenon that makes them unique. Although the writers are not affiliated with an organized movement, according to the passage, this is not the phenomenon that makes them unique.

A slumber did my spirit seal; I had no human fears: She seemed a thing that could not feel The touch of earthly years. No motion has she now, no force; She neither hears nor sees; Rolled round in earth's diurnal course, With rocks, and stones, and trees. —William Wordsworth, "A Slumber Did My Spirit Seal" The second stanza describes the author's belief that A.the person who died is now free from suffering B.the soul lives on after death C.someday he will be reunited with the person who died D.through death, the spirit becomes a part of nature

D.through death, the spirit becomes a part of nature Option (D) is correct. This question tests your ability to interpret the literal and figurative meanings of a poem. To paraphrase the second stanza, the person no longer moves, hears, or sees. The body is in the earth and is part of the daily movement of Earth around the sun, just like the rocks, stones, and trees. Therefore, in death the person's spirit has become a part of nature.


Related study sets

Chapter 21 - The immune system: Adaptive

View Set

Analyzing How Nuance Affects Meaning

View Set

IB Biology HL Section 11.3: The Kidney and Osmoregulation

View Set

Ch 1, Ch 5-20 Notes and Highlights

View Set

Health Assessment- PrepU Chapter 8

View Set

LPI Linux Essentials 010 V1.6 - Chapter 12 Quiz

View Set

Chapter 5: Job-based Structures and Job Evaluation

View Set